Задачи с ответами по банковскому праву: Главная — Северо-Западный институт (филиал) МГЮА

Задачи по хозяйственному праву с решением

Задачи по теме Правовое регулирование кредитных отношений.

Задача 60.

Между фирмой «Ш. ЛТД» и фирмой «РЛТД» был заключен договор, в соответствии с которым последняя обязалась в течение года со дня подписания договора погасить кредит, выданный фирме «Ш. ЛТД» Проминвестбанком. При этом Проминвестбанк не был поставлен в известность о заключенной сделке.

Правомерны ли действия сторон в этой ситуации? Какие последствия заключения такой сделки?

Решение

Указан. в условии договор имеет все признаки перевода долга. В соотв. со ст.201 ГК Украины, перевод должником своего долга на др. лицо допускается лишь с согласия кредитора. В дан. случае действия фирм неправомерны, сделку следует признать недействительной и требовать погашения кредита от фирмы «Ш-ЛТД».

Задача 62.

27.05.97 г. коммерческий банк (КБ) «Будбанк» и малое частное предприятие (МЧП) «Арио» заключили кредитный договор о предоставлении последнему кредита в сумме 6 млн. грн. со сроком погашения задолженности по кредиту 27.07.97 г. Своевременный возврат кредита был гарантирован арендным предприятием (АЛ) «Ремстройкомплект» путем направления в банк гарантийного письма от 01.06.97 г.

Кредитные средства не были своевременно возвращены в банк. Банком была направлена претензия в адрес гаранта — АЛ «Ремстройкомплект», однако в ней отсут­ствовали такие реквизиты, как дата предъявления претензии, номер претензии, подпись руководителя или его заместителя. 01.11.97 г. банк направил в суд исковое заявление о взыскании задолженности по кредитному договору.

Решением от 20.12.97 г. иск был удовлетворен. Свое решение судья мотивировал тем, что в соответствии со ст. 196 ПС Украины на гаранта распространяются правила ст. 191 ПС Украины, т. е. для гаранта устанавливается такой же правовой режим, как и для поручителя. Поэтому МЧП «Арио» и АП «Ремстройкомплект» должны солидарно отвечать за кредит и пропеты по нему.

Решение арбитражного суда от 20. 12.97 г. было отменено арбитражной надзорной коллегией Высшего Арбитражного Суда Украины.

Какие ошибки были допущены судом при разрешении дела? Какое решение должен принять суд по этому делу? Как должен заключаться договор гарантии?

Решение

Согласно кредитного договора, заключенного между коммерческим банком (КБ) «У..» и акционерным обществом (АО) «В…», последнему был предоставлен кредит в сумме 1,5 млн. грн. Согласно условий договора выплата процентов за пользование кредитом должна была производиться ежеквартально, начиная с четвертого месяца пользования кредитом. Однако банк через неделю с момента выдачи кредита начал досрочно в безакцептном порядке списывать с текущего счета акционерного общества суммы в счет погашения процентов за кредит. Правомерны ли действия банка?

Задачи по теме  Правовая защита банковской тайны

Задача 64.

Банком был получен запрос из П-го райотдела внутренних дел г. Одессы следующего содержания: «В связи с проведением доследственной проверки прошу предоставит информацию о движении денежных средств по текущему счету общества с ограниченно ответственностью «Н. ..».

Имеет ли право банк предоставить запрашиваемую информацию?

Решение

Банк имеет право предоставить запрошенную инф-цию. Ст.52 ЗУ «О банках и банковской деят-ти» предусматривает содержание банковской тайны. Справки по операциям и счетам юр. лиц выдаются самим юр. лицам, ГНИ по вопросам налогообложения, а также в случаях, предусмотренных закон-вом, судам по письменному требованию, органам прокуратуры, службы безопасности, внутренних дел, антимонопольному комитету.

Задача 65.

Государственной налоговой инспекцией по К-му р-ну г. Одессы был направлен запрос в коммерческий банк «О…» следующего содержания: «В связи с возникшей необходимостью прошу направить в наш адрес сведения о наличии счетов, открытых на имя гр-на Иванова А. А., и информацию о движении денежных средств по этим счетам».

Должен ли банк удовлетворить требования, содержащиеся в запросе?

Решение

Банк имеет право предоставить запрошенную инф-цию. Ст.52 ЗУ «О банках и банковской деят-ти» предусматривает содержание банковской тайны. Справки по операциям и счетам юр. лиц выдаются самим юр. лицам, ГНИ по вопросам налогообложения.

Задача 66.

В банк «Южный» был направлен запрос Ч-ской таможней с требованием предоставить информацию о движении денежных средств по текущему счету частного предприятия «А…».

Должен ли банк предоставил, информацию на подобный запрос?

Решение

В связи с тем, что требования не подкреплены какими либо сопроводительными документами банк отклонит требования о выдаче конфиденциальной информации о клиенте.

Задела 67.

27.08.98 г. Государственной налоговой администрацией Украины (ГНАУ) был принят приказ №415 «О порядке предоставления органам государственной налоговой службы учреждениями банков информации о наличии и движении средств на счетах клиентов (по требованию) и информации о проведении разовых денежных операций в больших размерах». Данным Приказом ГНАУ был утвержден Порядок, обязывающий Национальный Банк Украины и коммерческие банки предоставлять в электронном виде средствами электронной почты НБУ органам ГНА информацию о дебетовых и кредитовых оборотах и остатках средств на счетах всех клиентов банка. Также данным приказом устанавливалась периодичность предоставления такой информации и порядок формирования файлов с запрашиваемой информацией.

Постановлением Высшего арбитражного суда Украины от 30.04.99 г. приказ ГНАУ от 27.08.98 г. был отменен. Проанализируйте законность данного приказа ГНАУ. По каким основаниям он был отменен Высшим арбитражным судом Украины?

Задачи по теме Операции банков по осуществлению безналичных расчето

в

Задача 54.

На текущий счет предприятия «А…» в банке «К…» были направлены денежные средства частной фирмы «Б…» в качестве оплаты за оказанные услуги согласно договора. Поступившие денежные средства банк «К…», минуя текущий счет, зачислил ссудный счет предприятия «А. ..» для погашения задолженности по кредитному договор При этом на текущем счете предприятия «А…» числились неисполненными платежи поручения клиента о перечислении налогов в бюджет.

Правомерны ли действия банка в этой ситуации?

Решение

Действия банка в этой ситуации неправомерны (противоречат действующ. закон-ву). Положение о кредитовании предусматривает заключение кредитного договора м/д банком и субъектом хоз. деят-ти. Платежи со счета пр-я производятся банком с согласия владельца счета и в очередности, установленной руководителем пр-я. Без согласия владельца перечисление ср-в с его счета осущ-ся с целью взыскания недоимок по всем видам платежей в бюджет, в фонд соцстрах-я, др. внебюджетные фонды, штрафных платежей по распоряжению гос. КРС и Антимонопольного Комитета. При отсутствии договоренности об отсрочке погашения кредита банк имеет право на применение штрафных санкций в размерах, предусмотренных договором.

Задача 55.

Направленные обществом с ограниченной ответственностью (ООО) «М. ..» на текущий счет частного предприятия (ЧП) «В…» денежные средства были ошибочно зачислен банком на текущий счет Фирмы «А…». Обнаружив ошибку, банк направил зачисленную сумму на текущий счет ЧП «В…».

Правомерны ли действия банка? Каким образом осуществляется возврат ошибочно зачисленных денежных средств?

Решение

Действия банка правомерны. Возврат средств должен быть осуществлён аннулированием операции зачисления средств на счёт ЧП «В…»

Задача 56.

В банк было направлено постановление следователя о наложении ареста на денежные средства в сумме 20 000 грн. на текущем счете частной фирмы «Р…» в связи расследованием уголовного дела в отношении должностных лиц указанной фирмы и на оснований ст. ст. 29, 126 УПК Украины. В постановлении следователя также содержалось требование к банку перечислить арестованную сумму на депозитный счет УВД.

Законно ли постановление следователя? Какие действия обязан произвести банк после учения постановления о наложении ареста на денежные средства на текущем счете предприятия?

Решение

Постановление следователя незаконно. Банк должен заморозить текущий счёт фирмы «Р…» на время расследования уголовного дела.

Задача 57.

Государственной налоговой инспекцией П-го р-на г. Одессы было направлено в банк юсовое распоряжение для бесспорного списания денежных средств в сумме 2000 грн. с дополнительного текущего счета частного предприятия «С…». Остаток денежных средств на счете предприятия составлял 150 грн.

Какие действия должны быть произведены банком в этой ситуации?

Решение

Расчетные документы на бесспорное списание ср-в с доп. счетов принимаются банком к выполнению только при наличии на этих счетах ср-в в достаточном размере. В случае недостаточности ср-в для полного выполнения расчетных документов на списание ср-в осуществляется их частичное списание (п.145 с изменениями, внесенными согласно постановлениям Правления НБУ от 30.12.96 № 348 и от 02.06.97 № 173).

Задача 58.

15.08.98 г. банком «А…» было получено инкассовое распоряжение Государственной налоговой инспекции К-го р-на г. Одессы от 01.08.98 г. на бесспорное списание денежных средств в сумме 200 грн. с текущего счета общества с ограниченной ответственностью (ООО) «Н…». В назначении платежа было указано: «недоимка по налогу на прибыль; на основании ст. 11 Закона Украины «О государственной налоговой службе» и ст.6 Декрета Кабинета Министров Украины №8-93 «О взыскании невнесенных в срок налогов и неналоговых платежей».

Указанное инкассовое распоряжение не было принято банком к исполнению.

Правомерны ли действия банка?

Решение

Действия банка неправомерны. В соответствии со ст. 11 Закона Украины «О государственной налоговой службе» и ст.6 Декрета Кабинета Министров Украины №8-93 «О взыскании невнесенных в срок налогов и неналоговых платежей» банк обязан списать средства. На банк будет наложен штраф.

Задача 59.

В связи с непредставлением бухгалтерской отчетности в органы государственной налоговой службы частным предприятием (ЧП) «В. ..», Государственной налоговой инспекцией по С-му р-ну г. Одессы было выдано распоряжение о приостановлении операций на всех счетах ЧП «В…».

Какие операции могут производиться по счетам предприятия в этом случае?

Решение

Выплаты налоговых отчислений, отчислений в фонд зарплаты и другие социальные фонды.

 

Задачи по теме Последствия неисполнения и ненадлежащего исполнения внешнеэкономических контрактов

Задачи 41.

Украинское предприятие занимается поставкой в Украину автозапчастей по контрактам с фирмами Германии и Турции. По материалам правоохранительных органов установлено, что одну треть поставок это предприятие осуществляет контрабандным путем, без уплаты налогов и таможенных пошлин. Какие меры могут быть применены к предприятию? Может ли МВЭСторг применить санкции к предприятию по своей инициативе?

Решение

В соотс. со ст 4 ЗУ «О едином таможенном тарифе» товары и др. ввозимые предметы, ввозимые на тамож. террит. Украины и вывозимые за пределы этой террит., облагаются тамож. пошлиной. Перемещение через тамож террит. Украины вне тамож. контроля или со скрытием от тамож. контроля товаров, валюты, ценностей и др. предметов в больших размерах (общая стоимость предметов – 500 и более официально установленных минимальных размеров з/п) считается контрабандой (ст. 100 Тамож. кодекса Украины). В случае осуществления контрабандных поставок, действия дирекции пр-я квалифицируются статьей 70 УК Украины (наказание – лишение свободы на срок от 3 до 5 лет с конфискацией предметов контрабанды и имущества; при повторном осуществлении – лишение свободы на срок от 5 до 10 лет с конфискацией имущества и предметов контрабанды).

Задача 43.

Английская фирма «Salex» дважды ввозила в Украину по контракту компьютеры и передавала украинским малым предприятиям. Налоги фирма «Salex» не платила, мотивируя это тем, что ввозит оборудование в благотворительных целях.

По представлению налоговых органов МВЭСторг принял решение о применении индивидуального режима лицензирования к фирме «Salex». Через две недели фирма «Salex» погасила все налоговые задолженности.

Правильно ли применена санкция к фирме «Salex»?

Может ли фирма «Salex» далее заниматься внешнеэкономической деятельностью по своему усмотрению?

Какие действия должна совершить фирма «Salex» для продолжения внешнеэкономической деятельности?

Решение

Санкция к анг. фирме применена неправильно. По ст.37 «Спец. санкции при нарушении настоящего и связанного с ним законов Украины» ЗУ «О ВЭД» с дан. фирмы следовало взыскать штраф за неисполнение налоговых обязательств. Согласно ст.8 «Тамож. ст-ть» ЗУ «О едином тамож. тарифе», ст.4 ЗУ «О НДС» фирма «Salex» должна была уплатить в бюджет: тамож. пошлину в полн. объеме ставки ввозн. тамож. пошлины, предусмотренной единым тамож. тарифом Украины; НДС; тамож. сборы. В соотв. с ЗУ «О ВЭД» в случае неисполнения налоговых обязательств на деят-ть субъекта ВЭД налагается штраф. Санкции применяются на срок до устранения субъектами ВЭД нарушений действ. закон-ва в сфере ВЭД, поэтому фирма «Salex», погасив свои налоговые обязательства, может продолжать свою деят-ть.

Задача 44.

Общество с ограниченной ответственностью (ООО) (г. Николаев) в г. Стамбуле (Турция) заключило контракт с турецкой фирмой «Marmara» на поставку ткани на сумму 10 000 долларов США.

Однако турецкая фирма не выполнила контрактные обязательства, мотивируя это произошедшим землетрясением.

ООО предъявило претензию к фирме «Marmara», потребовав возмещения убытков в сумме 13 000 долларов США по причине того, что из-за непоставки ткани был сорван договор с «Укрторгом» о реализации швейной продукции и ООО был уплачен штраф «Укрторгу» за неисполнение контракта.

ООО в претензии утверждает, что фирма «Marmara» расположена в местности, не пострадавшей от землетрясения, и имела реальную возможность выполнения кон­трактных обязательств.

Правомерна ли претензия ООО?

Правильно ли ООО определило размер убытков?

Может ли ООО обратиться в международный арбитраж для разрешения спора?

Где и по праву какой страны будет решаться спор?

Какое решение может быть принято?

Решение

Согласно ст. 6 ЗУ «О ВЭД» претензия ООО правомерна. Договор может быть признан недействительным в суд. порядке, если он не соответствует требованиям ЗУ или Межд. договоров Украины. Размер убытков (возмещение) опр-ся в %-ах от ст-ти недопоставленного товара (суммы неуплач. ср-в). Права и обязанности сторон ВЭД опр-ся правом страны, выбранной сторонами при заключении договора (или в результате дальнейш. согласования). Форс-мажорные обстоятельства должны быть предусмотрены в п. 1.10 ВЭ контракта, срок действия которых подтверждается Торгово-пром. палатой соотв. страны. К правам и обязанностям по дан. ВЭК применяется право Турции, где учреждена фирма «Marmara», осуществляющая выполнение такого контракта. ООО (г. Николаев) правильно определило размер убытков, т. к. был сорван договор с «Укрторгом», а также может обратиться в межд. арбитражн. суд, если это предусмотрено условиями ВЭК.

Задача 45.

Французская фирма в Вене (Австрия) заключила контракт лизинга трех грузовых автомашин с иранским предприятием, однако уже при ввозе автомашин в Украину украинское предприятие задекларировало их как свою собственность и в течение двух лет не производило платежей французской фирме.

Фирма обратилась в международный арбитраж за решением спора.

В ходе арбитражного процесса фирма обратилась к украинской стороне с предложением о заключении мирового соглашения в случае уплаты его контрактных задолженностей.

Где и по праву какой страны может решаться данный спор

Какой порядок обращения истца в международный арбитраж?

В случае согласия украинской стороны каким образом будет происходить заключение мирового соглашения арбитраже?

В случае несогласия украинской стороны и неуплаты ею долгов, какой порядок дальнейшего рассмотрения спора в международном арбитраже?

Решение

Спор должен решаться во Франции по французскому праву, поскольку автомашины принадлежат французской фирме. Действия истца на Украине: обращение в районный хозяйственный суд – пересылка дела в международный арбитражный суд – французский хозяйственный суд.

После выплаты контрактных задолженностей украинская фирма должна подписать контракт по использованию или лизингу автомашин с французской фирмой.

В случая неуплаты долгов французская фирма вправе требовать от украинской фирмы возврата имущества и выплаты штрафа.

Задача 46.

Общество с ограниченной ответственностью (ООО) (г. Донецк) и фирма «Ален Мак» (Болгария) заключили в г. Варна (Болгария) контракт о поставке парфюмерии. Контрактные обязательства не были выполнены по вине болгарской стороны.

ООО обратилось в Международный коммерческий арбитражный суд при Торгово-Промышленной палате Украины за разрешением спора.

Болгарская сторона не представила в установленный срок возражения на иск и не явилась по вызову в заседание арбитража

Арбитр единоличным решением отложил рассмотрение данного спора в связи с неявкой ответчика.

Правомерно ли рассмотрение данного спора в указанном арбитраже? Мог ли арбитр единолично пригашать решение об отсрочке рассмотрения спора? Законно ли принятое арбитром решение?

Может ли болгарская сторона оспорить данное решение международного арбитража? Если да, то в каком порядке и в какой орган?

Решение

Правомерно.

Решение, принятое арбитром правомерно. В случае неявки одной из сторон два раза рассмотрение спора откладывается. Болгарская сторона оспорить данное решение арбитра не может.

Задача 47.

Морская арбитражная комиссия при Торгово-промышленной палате Украины рассматривала спор по иску украинского ООО к норвежской фирме «Gals» за неуп­лату денег за проданное судно.

Во время арбитражного производства выяснилось, что истец (ООО) объявлено банкротом и прекратило свое существование.

Может ли арбитраж прекратить производство по настоящему делу при данных обстоятельствах?

Если да, то какой порядок прекращения арбитражного производства по делу?

Если нет, то какой дальнейший порядок арбитражного рассмотрения спора?

Решение

Нет. Норвежская фирма должна выплатить деньги за судно. Полученную сумму денег суд должен распределить между государством (оплата платежей) и бывшими акционерами в соответствии с их долей.

 

Задачи по теме Специальные договоры в сфере купли-продажи

Задача 28.

При оплате угля резидентом-! (импортером газа) на резидента-2 (покупателя угля у резидента-1) при согласии и в пользу нерезидента-! (поставщика газа резидсшу-1), а в последующем совершена уступка нерезидентам-! нерезнденту-2 требования от резидента-2 по оплате газа (оплата осуществлена в иностранной валюте).

Возможен ли перевод долга в данной ситуации?

Сведения об образовательной организации

Размер:

A

A

A

Цвет: CCC

Изображения Вкл.Выкл.

Обычная версия сайта

Горно-Алтайский государственный университет

  • Университет
    • Обращение ректора
    • История
    • Ученый совет
    • Администрация
    • Интеллект-центр
    • Медиацентр
    • Отдел делопроизводства
    • ОПОД и УИК
    • Управление бухгалтерского учета и финансового контроля
    • Планово-финансовое управление
    • Управление кадров
    • Центр цифрового развития
    • Административно-хозяйственное и материально-техническое подразделение
    • Контрактный управляющий
    • Противодействие коррупции
    • Сведения о доходах
    • Антитеррористическая безопасность
    • Международная деятельность
    • Безопасность и охрана труда
    • Лучшие студенты
    • Структура
    • Календарь мероприятий
    • Профком студентов и аспирантов
    • Республиканская профсоюзная организация высшей школы
    • Вопросы ректору
  • Образование
    • Факультеты и институт
    • Учебно-методическое управление
    • Методический совет ГАГУ
    • Образовательная деятельность
    • Отдел практической подготовки студентов
    • Заочное обучение
    • Центр дополнительного образования
    • Центр карьеры
    • Методические и иные документы
    • Консультационный центр поддержки студентов
    • Региональный центр финансовой грамотности
    • Учебно-тренинговый центр
    • Центр развития педагогического образования
    • Локальный центр тестирования иностранных граждан
  • Воспитание
    • Центр воспитательной и внеучебной работы
    • Центр социально-психологической помощи
    • Совет по воспитательной работе
    • Волонтёрский центр
    • Cовет обучающихся
    • Информационные материалы
    • Совет кураторов
    • Клуб выпускников
  • Наука
    • Новости науки
    • Центр развития науки и инноваций
    • Отдел научно-технической информации
    • Отдел подготовки научно-педагогических кадров
    • Библиотечно-издательский центр
    • Лаборатории, НШ, НИЦ, вузовско-академическая кафедра
    • Музейный комплекс ГАГУ
    • Научные мероприятия в ГАГУ
    • Центр развития туризма и гостеприимства
    • Национальный проект «Наука и университеты»
  • Культура и спорт
    • Немецкий культурный центр
    • Центр языка и культуры Китая
    • Туристский клуб «Горизонт»
    • Спортивный клуб «Буревестник»
    • Киберспорт
    • Спортивные достижения студентов и сотрудников ГАГУ
    • Военно-патриотический клуб «БАРС»
    • Спортивно-оздоровительная база на Телецком озере
  • Контакты и адреса
    • Телефонный справочник
    • Платежные реквизиты
    • Символика ГАГУ
    • Карта корпусов
    • Карта сайта
  • Сведения об образовательной организации
  • Файлы

Банковское право — Юридическая библиотека штата Висконсин

Перейти к связанным темам, библиотечным ресурсам и статьям с обзорами законов

  • Агентства/организации
    • Сеть по борьбе с финансовыми преступлениями (Министерство финансов США)
      Сеть правоохранительных органов, созданная для борьбы с отмыванием денег и внутренними и международными финансовыми преступлениями. Веб-сайт предлагает доступ к положениям Закона о банковской тайне, руководствам, часто задаваемым вопросам, формам и информации о регистрации; принудительные меры за нарушения ЗБТ; постановления, рекомендации и отчеты FinCEN; и более.
    • Федеральная корпорация страхования депозитов (FDIC)
    • WI Департамент финансовых учреждений
  • Ответы на вопросы о банковских счетах и ​​банках (Министерство финансов США)
    Банковские сборы и сборы за овердрафт, процедуры, кредитные карты, кредиты, чеки и многое другое.
  • Банковские данные (Федеральный Корпорация страхования депозитов)
    Информация об отдельных банках и банковской отрасли. Определить статус застрахованного депозитного учреждения, их финансовое состояние и их состояние относительно других банков.
  • Чеки/расчетные счета
    • Ответы о банковских счетах и ​​банках (Министерство финансов США)
      Банковские сборы и комиссии за овердрафт, процедуры, кредитные карты, кредиты, чеки и многое другое.
    • Соответствие С Положением CC, Наличие средств и инкассация чеков (Федеральный Совет резерва)
    • Чеки, помеченные как платеж в полном объеме (Департамент финансовых учреждений штата Висконсин)
      Обсуждается юридический статус заявления о полной оплате.
    • Плохие/бесполезные чеки
      • Мошенничество с чеками: Руководство по предотвращению убытков (Министерство финансов США)
      • Бесполезный Чеки (Департамент финансовых учреждений штата Висконсин)
      • Бланки и справочники бесполезных чеков округов Висконсин (Юридическая библиотека штата Висконсин)
      • Устав штата Висконсин: с. 403.414(7) Ответственность
      • Устав штата Висконсин: с. 422.202 Сборы
      • Висконсин Устав: с. 943.24 — 943.245 «… Бесполезные чеки»
  • Жалобы
    • Жалобы на банки и финансовые учреждения (Совет Федеральной резервной системы)
    • Жалобы потребителей (Управление валютного контролера США)
    • Жалобы на федеральные или государственные кредитные союзы (Национальная администрация кредитных союзов)
    • Финансовые учреждения / Жалобы на бизнес (Департамент финансовых учреждений штата Висконсин)
    • Ипотечный банк (Департамент финансовых учреждений штата Висконсин)
  • Каталоги
    • WI Banks & Savings Institutions (Департамент финансовых учреждений WI)
    • Кредитные союзы штата Висконсин (Департамент финансовых учреждений штата Висконсин)
  • Ошибка Банки (FDIC)
    «Узнайте, как на счета и кредиты влияет банкротство банка, как искать невостребованные средства и как получить освобождение от залога». Включает в себя Список несостоятельных банков, Дивиденды FDIC от несостоятельных банков, историческая статистика и краткие описания банкротств каждого банка с 1991 года.
  • Формы
    • Образцы форм (Wisconsin Credit Association)
      Включает кредитные заявки, образец письма о подделке чека, гарантийные письма, вексель, доказательство претензии.

  • Ссуды до зарплаты / отложенные депозиты
    • Кредиторы до зарплаты (Департамент финансовых учреждений штата Висконсин)
    • Устав о кредитовании до зарплаты (Национальная конференция законодательных собраний штатов)
    • День выплаты жалованья Ссуды (Федеральная торговая комиссия)
      Объяснение и альтернативы ссудам до зарплаты. Инструкции о том, как подать жалоба.
    • Ссуды до зарплаты (Департамент финансовых учреждений штата Висконсин)

Закон (Дополнительные положения, правила и мнения могут относиться к вашей конкретной ситуации. )

  • Устав штата Висконсин: гл. 138 «Деньги и процентные ставки»
  • Устав штата Висконсин: гл. 186 «Кредитные союзы»
  • Устав штата Висконсин: гл. 214-224 «…Банки; Сберегательные общества»
  • Устав штата Висконсин: гл. 404 «Единый коммерческий кодекс — Банковские депозиты и инкассо»
  • Устав штата Висконсин: гл. 410 «Единый коммерческий кодекс — Денежные переводы»
  • Висконсин Админ. Код: ДФИ-Бкг 3-80 «Банковское дело»
  • Висконсин Админ. Код: DFI-CU 51-72 «Кредитные союзы»
  • Висконсин Админ. Код: ДФИ-СБ 1-22 «Сберегательные кассы»
  • Висконсин Админ. Код: DFI-SL 1-21 «Сбережения и кредиты»
  • Устав штата Висконсин и Административный кодекс банковского дела, кредитных союзов, финансовых услуг, сберегательных учреждений (Департамент финансовых учреждений штата Висконсин)
  • Код США: 12 USC «Банки и банковское дело»
  • CFR: 12 CFR «Банки и банковское дело»

Узнать больше уставы, положения и заключения.

Resources County

Рекомендуемые ссылки из нашей базы данных ресурсов округа:

  • Руководства
    • Процедура для обработки достоверных чеков (Окружной адвокат IOWA)
    • БЕЛЛАННЫЕ ЧЕЛОВЕКА (IOWA DIASTEOREY)
    • .0018

    Задайте вопрос

    Связанные темы

    • Кредит и долг
    • Валюта, юридический платеж, биткойны и многое другое
    • Торта личности
    • Legal Assive

    Art.tral Artft

    • 5 40005 40005. The Payday Loan Problem in Wisconsin
      2006 Wis. L. Rev. 1627
    • Wisconsin’s New Mortgage Banking Law
      Wis. Lawyer 62 n3 (март 1989 г.) p15

    Ресурсы библиотеки

    • Banking & Financial Institutions Law in a Nutshell
    • Banking & Lending Institution Forms: with Commentary and Checklist
    • Banking Law and Regulation
    • Specialized Legal Research

    Journals & Legal Databases

    • Banking Law Journal
    • Fordham Журнал корпоративного и финансового права
    • HeinOnline
    • Банковский институт Северной Каролины

    Учебный центр

    • Найм юриста
    • Поиск форм
    • Введение в закон
    • Начните здесь: Руководство по кредитам и долгам

    50 лучших вопросов и ответов на собеседовании в банковской сфере (2022)

    как опытные кандидаты, чтобы получить работу своей мечты.

    👉 Бесплатная загрузка в формате PDF: Банковское интервью Вопросы

    1) Что такое банк? Какие есть виды банков?

    Банк – это финансовое учреждение, имеющее лицензию получателя денежных вкладов. Есть два типа банков, коммерческих банков и инвестиционных банков. В большинстве стран банки регулируются национальным правительством или центральным банком.


    2) Что такое инвестиционно-банковская деятельность?

    Инвестиционно-банковская деятельность управляет портфелями финансовых активов, товаров и валюты, фиксированным доходом, корпоративными финансами, корпоративными консультационными услугами по слияниям и поглощениям, выпиской долговых обязательств и акционерного капитала и т. д.


    3) Что такое коммерческий банк?

    Коммерческий банк принадлежит группе лиц или члену Федеральной резервной системы. Коммерческий банк предлагает услуги физическим лицам, они в первую очередь занимаются приемом депозитов и кредитованием бизнеса. Такой банк зарабатывает деньги, налагая проценты на кредит, взятый заемщиком. Деньги, внесенные клиентом, будут использоваться банком для предоставления бизнес-кредитов, автокредитов, ипотечных кредитов и кредитов на ремонт дома.

    Банковское интервью Вопросы

    4) Каковы типы коммерческих банков?

    Ниже приведены типы коммерческих банков. банковское дело

    Корпоративные банковские операции, связанные с управлением денежными средствами, андеррайтингом, финансированием и выпуском акций и облигаций

    c) Ценные бумаги и инвестиционно-банковские услуги

    Инвестиционно-банковские услуги управляют портфелями финансовых активов, товаров и валюты, фиксированным доходом, корпоративными финансами, корпоративными консультационными услугами по вопросам слияний и поглощений, выпиской долговых обязательств и акций и т. д.

    d) Нет -традиционные варианты

    Существует множество небанковских организаций, которые предлагают финансовые услуги, подобные банковским. К субъектам относятся компании, выпускающие кредитные карты, агентства по составлению отчетов о кредитных картах и ​​эмитенты кредитных карт 9. 0003


    5) Что такое потребительский банк?

    Потребительский банк — это новое дополнение в банковском секторе, такие банки существуют только в таких странах, как США и Германия. Этот банк предоставляет своим клиентам кредиты на покупку телевизора, автомобиля, мебели и т. д. и дает возможность легкой оплаты в рассрочку.


    6) Какие бывают виды счетов в банках?

    a) Текущий счет: Вы можете получить доступ к счету как к сберегательному счету, но, в отличие от сберегательного счета, вы не можете получать проценты по этому счету. Преимущество этой учетной записи в том, что нет ограничений на вывод средств.

    б) Сберегательный счет: Вы можете хранить свои деньги на таком счете, а также получать проценты по нему. Количество выводов ограничено и необходимо поддерживать минимальную сумму баланса на счете, чтобы оставаться активным.

    c) Счет денежного рынка: Этот счет дает преимущества как сберегательного, так и расчетного счета. Вы можете снять сумму, и все же вы можете заработать на ней более высокие проценты. Этот счет можно открыть с минимальным балансом.

    d) CD (Депозитный сертификат) Счет: На такой счет вы должны положить свои деньги на определенный период времени (5-7 лет), и вы будете получать проценты по ним. Процентная ставка определяется банком, и вы не можете снять средства, пока не истечет установленный период.


    7) Как вы можете управлять своими счетами?

    Вы можете управлять своими банковскими счетами различными способами, например,

    а) Интернет-банкинг

    б) Банкинг по телефону или мобильному телефону

    в) Отделение или безналичный расчет

    d) Банкомат (банкомат)


    8) На что следует обратить внимание перед открытием банковских счетов?

    Перед открытием банковского счета, если это сберегательный счет, вы должны проверить процентную ставку по вкладу и ее постоянство в течение периода. Если у вас есть текущий счет, посмотрите, сколько чеков можно использовать бесплатно. Некоторые банки могут взимать плату за использование бумажных чеков или заказ новых чековых книжек. Кроме того, проверьте различные варианты дебетовых карт, которые предоставляются при открытии счета и функциях онлайн-банкинга.


    9) Что такое перекрестный чек?

    Перечеркнутый чек указывает на то, что сумма должна быть зачислена на счет получателя и не может быть обналичена банком в кассе. Здесь, на изображении № 2, вы можете видеть две пересекающиеся линии в левом углу чека, которые указывают на пересечение чека.


    10) Что такое защита от овердрафта?

    Защита от овердрафта — это услуга, которую банк предоставляет своему клиенту. Например, если у вас два счета, сберегательный и кредитный, в одном и том же банке. Теперь, если на одном из ваших счетов недостаточно наличных для обработки чеков или для покрытия покупок. Банк будет переводить деньги с одного счета на другой счет, на котором нет наличных денег, чтобы предотвратить возврат чека или оплатить ваши покупки или счета за электричество.


    11) Взимает ли банк плату за услугу «защита от овердрафта»?

    Да, банк будет взимать плату за услуги «защиты от овердрафта», но плата будет применяться только тогда, когда вы начнете пользоваться услугой.


    12) Что такое (годовая) годовая процентная ставка?

    APR означает годовую процентную ставку, и это плата или проценты, которые банк налагает на своих клиентов за использование их услуг, таких как кредиты, кредитные карты, ипотечный кредит и т. д. Процентная ставка или налагаемые сборы рассчитываются ежегодно.


    13) Что такое «основная ставка»?

    По сути, «основная ставка» — это процентная ставка, устанавливаемая крупнейшими банками страны (США) для своих привилегированных клиентов, имеющих хороший кредитный рейтинг. Большая часть «переменных» процентов зависит от «основных ставок». Например, «годовая процентная ставка» (годовая процентная ставка) по кредитной карте составляет 10% плюс основная ставка, а если основная ставка составляет 3%, текущая «годовая ставка» по этой кредитной карте будет составлять 13%.


    14) Что такое «фиксированная» и «переменная» годовая процентная ставка?

    «Годовая процентная ставка» может быть «фиксированной» или «переменной». В «Фиксированной годовой процентной ставке» процентная ставка остается неизменной на протяжении всего срока кредита или ипотеки, в то время как в «Переменной годовой процентной ставке» процентная ставка будет меняться без предварительного уведомления в зависимости от других факторов, таких как «основная ставка».


    15) Какие типы приложений банковского программного обеспечения доступны в отрасли?

    Существует множество типов банковских приложений, и некоторые из них перечислены ниже

    а) Система интернет-банкинга: Интернет-банкинг позволяет клиентам и финансовым учреждениям проводить окончательную транзакцию с использованием банков или веб-сайтов финансовых учреждений.

    b) Банкомат (банкомат): Это электронное банковское отделение, которое позволяет клиентам совершать основные операции.

    c) Базовая банковская система: Базовая банковская система – это услуга, предоставляемая объединенными в сеть отделениями банка. При этом клиент может снимать деньги в любом отделении.

    d) Система управления кредитами: База данных собирает всю информацию и отслеживает клиентов, которые занимают деньги.

    e) Система управления кредитами: Система управления кредитами – это система управления кредитными счетами, оценки рисков и определения суммы кредита, которую можно предложить клиенту.

    f) Система управления инвестициями: Это процесс управления деньгами, включая инвестиции, банковское дело, составление бюджета и налоги.

    g) Система управления фондовым рынком: Управление фондовым рынком — это система, которая управляет финансовым портфелем, таким как ценные бумаги и облигации.

    h) Система управления финансами: Система управления финансами используется для управления и учета доходов, расходов и активов, а также для учета прибыли.


    16) Какова «стоимость долга»?

    Когда какая-либо компания занимает средства у финансового учреждения (банка) или других ресурсов, проценты, выплачиваемые на эту сумму, называются «стоимостью долга».


    17) Что такое «баллонный платеж»?

    Единовременный платеж — это последний причитающийся единовременный платеж. Когда весь платеж по кредиту не амортизируется в течение срока кредита, оставшаяся часть должна быть окончательным погашением кредитору. Воздушный платеж может происходить в рамках ипотеки с регулируемой или фиксированной ставкой.


    18) Что такое «Амортизация»?

    Погашение кредита в рассрочку для покрытия основной суммы с процентами называется «Амортизация».


    19) Что такое отрицательная амортизация?

    Когда погашение кредита меньше суммы накопленных процентов по кредиту, происходит отрицательная амортизация. Это увеличит сумму кредита, а не уменьшить его. Он также известен как «отложенный процент».


    20) В чем разница между «чеком» и «траттой до востребования»?

    Оба используются для перевода суммы между двумя счетами одного или другого банка. «Чек» выдается лицом, у которого есть счет в банке, а «Вексель до востребования» выдается банком по запросу и взимает плату за услугу. Кроме того, тратта до востребования не может быть аннулирована, в то время как чеки могут быть аннулированы после выдачи.


    21) Что такое отношение долга к доходу?

    Отношение долга к доходу рассчитывается путем деления общей суммы долга заявителя на его валовой доход.


    22) Что такое корректировочный кредит?

    Корректирующий кредит — это краткосрочная ссуда, предоставляемая Федеральным резервным банком (США) коммерческому банку для выполнения резервных требований и поддержки краткосрочного кредитования в случае нехватки денежных средств.


    23) Что вы подразумеваете под «зарубежным проектом»?

    Иностранная тратта является альтернативой иностранной валюте; он обычно используется для отправки денег в другую страну. Его можно приобрести в коммерческих банках, и они будут взимать плату в соответствии с правилами и нормами своих банков. Люди выбирают «иностранный перевод» для отправки денег, поскольку этот способ отправки денег дешевле и безопаснее. Это также позволяет получателю получить доступ к средствам быстрее, чем чек или денежный перевод.


    24) Что такое «кредитный рейтинг»?

    Классификация кредита на основе различных рисков и параметров, таких как риск погашения, кредитная история заемщика и т. д., известна как «классификация кредита». Эта система размещает ссуду по одной-шести категориям в зависимости от стабильности и риска, связанного с ссудой.


    25) Что такое «кредит-неттинг»?

    Система сокращения количества проверок кредитоспособности финансовых транзакций известна как кредит-неттинг. Такое соглашение обычно заключается между крупными банками и другими финансовыми учреждениями. Он помещает все будущие и текущие транзакции в одно соглашение, устраняя необходимость проверки кредитоспособности каждой транзакции.


    26) Что такое «Проверка кредитоспособности»?

    Проверка кредитоспособности или кредитный отчет выполняются банком на основе финансового кредита физического лица. Это делается для того, чтобы убедиться, что физическое лицо достаточно способно выполнить финансовые обязательства для своего бизнеса или любой другой денежной операции. Проверка кредитоспособности выполняется с учетом нескольких аспектов, таких как ваши обязательства, активы, доход и т. д.


    27) Что такое межбанковский депозит?

    Любой депозит, удерживаемый одним банком для другого банка, называется межбанковским депозитом. Банк, в котором находится депозит, называется банком-корреспондентом.


    28) Что такое ILOC (безотзывный аккредитив)?

    Это аккредитив или договорное соглашение между финансовым институтом (банком) и стороной, которой вручается письмо. Письмо ILOC не может быть аннулировано ни при каких обстоятельствах и гарантирует оплату стороне.

    Требуется, чтобы банк платил по траттам, отвечающим всем условиям ILOC. Он действует до указанного срока. Например, если малый бизнес хотел заключить контракт с зарубежным поставщиком на конкретный товар, они пришли бы к соглашению об условиях продажи, таких как стандарты качества и цены, и попросили бы свои соответствующие банки открыть аккредитив для сделки. .

    Банк покупателя направит аккредитив в банк продавца, где будут окончательно согласованы условия платежа и произведена отгрузка.


    29) В чем разница между банковской гарантией и аккредитивом?

    Между банковской гарантией и аккредитивом нет большой разницы, поскольку они оба предполагают платежное обязательство. Банковская гарантия несет в себе больший риск для банка, чем аккредитив, поскольку она защищает как покупателя, так и продавца.


    30) Что такое кассовый чек?

    Кассовый чек выдается банком от имени клиента и принимает гарантию платежа. Оплата производится из собственных средств банка и подписывается кассиром. Кассовый чек выдается при необходимости быстрого расчета.


    31) Что вы подразумеваете под со-творцом?

    Лицо, которое подписывает записку, гарантирующую выплату кредита от имени основного заявителя на получение кредита, называется созаемщиком или поручителем.


    32) Что такое ипотечный кредит?

    Кредит под залог дома, также известный как вторая ипотека, позволяет вам занимать деньги под залог стоимости вашего дома. Например, если стоимость дома составляет 1 50 000 долларов, и вы заплатили 50 000 долларов. Остаток задолженности по ипотеке составляет 100 000 долларов. Сумма в размере 50 000 долларов США представляет собой собственный капитал, представляющий собой разницу между фактической стоимостью дома и суммой, которую вы должны банку. На основе справедливости кредитор даст вам кредит.

    Обычно заявитель получает 85% кредита от собственного капитала, учитывая ваш доход и кредитный рейтинг. В этом случае вы получите 85% от 50 000 долларов, что составляет 42 500 долларов.


    33) Что такое кредитная линия?

    Кредитная линия представляет собой соглашение или договоренность между банком и заемщиком о предоставлении определенной суммы кредита по требованию заемщика. Заемщик может снять сумму в любой момент времени и оплатить проценты только на снятую сумму. Например, если у вас есть кредитная линия на 5000 долларов, вы можете снять всю сумму или любую сумму меньше 5000 долларов (скажем, 2000 долларов) и заплатить только проценты за снятую сумму (в данном случае 2000 долларов).


    34) Как банк получает прибыль?

    Банк получает прибыль различными способами

    a) Банковская цепочка создания стоимости

    b) Прием депозитов

    c) Предоставление средств заемщикам под проценты

    d) Процентный спред

    e) Дополнительные расходы на такие услуги, как ведение расчетного счета , онлайн-оплата счетов, транзакция через банкомат


    35) Что такое зарплатные карты?

    Карты для расчета заработной платы — это типы смарт-карт, выпускаемые банками для облегчения выплаты заработной платы между работодателем и работниками. Через зарплатную карту работодатель может зачислять выплаты заработной платы на смарт-карту работника, а работник может снимать зарплату, даже если у него нет счета в банке.


    36) Что такое платежи по карте?

    Существует два типа платежей картой

    a) Кредитная карта

    b) Дебетовая карта


    37) Что означает ACH?

    ACH расшифровывается как Automated Clearing House, который представляет собой электронный перевод средств между банками или финансовыми учреждениями.


    38) Что такое «плавающий запас доступности»?

    Резервный запас — это разница во времени между внесенными депозитами и фактически доступными средствами на счете. Пришло время обработать физический чек на ваш счет.

    Например, у вас уже есть 20 000 долларов США на вашем счету, и чек на еще 10 000 долларов США депонирован на ваш счет, но на вашем счете будет отображаться баланс в размере 20 000 долларов США вместо 30 000 долларов США до тех пор, пока ваш чек на 10 000 долларов США не будет очищен. Это время обработки называется запасом доступности. .


    39) Что вы подразумеваете под терминами «срок погашения кредита» и «доходность»?

    Дата, на которую наступает срок погашения основной суммы кредита, называется «сроком погашения кредита». Доход обычно называют дивидендом, процентом или доходом, который инвестор получает от ценных бумаг, таких как акции или облигации, проценты по срочному депозиту и т. д. Например, любые инвестиции в размере 10 000 долларов США при процентной ставке 4,25% дадут вам доход в размере 425 долларов США.


    40) Что такое индекс стоимости фондов (COFI)?

    COFI — это индекс, который используется для определения процентных ставок или изменений процентных ставок для определенных видов кредитов.


    41) Что такое оговорка о конвертируемости?

    Для определенного кредита существует положение о том, что заемщик может изменить процентную ставку с фиксированной на переменную и наоборот, что называется оговоркой о конвертируемости.


    42) Что такое списание?

    Списание — заявление кредитора заемщику о невыплате оставшейся суммы, когда заемщик сильно залез в долг. Неуплаченная сумма погашается как безнадежный долг.


    43) Что означает «LIBOR»?

    «LIBOR» означает лондонскую межбанковскую ставку предложения. Как следует из названия, это средняя процентная ставка, предлагаемая для долларов США или евро, депонированных между группами лондонских банков. Это международная процентная ставка, которая соответствует мировым экономическим условиям и используется банками в качестве базовой ставки для установления процентной ставки. LIBOR имеет 8 сроков погашения от овернайт до 12 месяцев и в 5 различных валютах. Один раз в день LIBOR объявляет свою процентную ставку.


    44) Что вы подразумеваете под термином «ростовщичество»?

    Когда ссуда незаконно взимается с высокой процентной ставкой, это называется «ростовщичеством». Ставки ростовщичества обычно устанавливаются законом штата.


    45) Что такое кредит до зарплаты?

    Ссуда ​​до зарплаты, как правило, небольшая сумма и краткосрочный кредит с высокой процентной ставкой. Заемщик обычно выписывает кредитору чеки с задним числом в отношении суммы, которую они хотят занять.


    46) Что вы подразумеваете под «подтверждением чека»?

    «Подтверждающий чек» гарантирует, что чек будет зачислен только на ваш счет. Это сводит к минимуму риск кражи. Обычно, подтверждая чек, кассир попросит вас расписаться на обратной стороне чека. Подпись должна соответствовать получателю. На изображении здесь показан подтвержденный чек.


    47) Какие виды кредитов предлагаются банками?

    Различные виды кредитов, предлагаемых банками:

    a) Необеспеченный личный кредит

    b) Личный кредит под залог

    c) Автокредиты

    d) Ипотечные кредиты

    e) Кредиты для малого бизнеса


    48) Какие существуют типы «срочных депозитов»?

    Существует два различных типа «Срочных депозитов»

    Специальные срочные депозиты : В этом типе «Срочных депозитов» заработанные проценты по депозиту добавляются к основной сумме и ежеквартально начисляются. Эта сумма накапливается и погашается вместе с основной суммой по истечении срока депозита.

    Обычные срочные депозиты : В этом типе «срочных депозитов» заработанный кредит зачисляется на счет инвестора один раз в квартал. В некоторых случаях проценты могут начисляться ежемесячно.

    Проценты по срочным депозитам не облагаются налогом. Вы также можете взять кредит под ваш фиксированный депозит.


    49) Какие виды кредитов предлагаются коммерческими банками?

    Ссуды

    Ссуды для стартапа

    Этот тип ссуды предлагается заемщику, чтобы начать свой бизнес, и может быть использован для создания витрины, для приобретения инвентаря или оплаты франшизы, чтобы запустить бизнес.

    Кредитная линия

    Кредитная линия — это еще один тип бизнес-кредита, предоставляемого коммерческими банками. Это больше похоже на безопасность вашего бизнеса; банк позволяет клиенту снять сумму из легкодоступных средств в неблагоприятный момент. Клиент или компания могут со временем вернуть деньги и снова снять деньги, не вдаваясь в процесс кредита.

    Кредиты для управления малым бизнесом

    Это Федеральное агентство (США), которое предоставляет финансирование малым предприятиям и предпринимателям. Кредиты SBA (Small Business Administration) выдаются через банки, кредитные союзы и другие кредиторы, сотрудничающие с SBA.


    50) Что такое «Скидка по счету»?

    «Скидка по счету» — это расчет по счету, когда ваш счет за электричество или газ продается банку для досрочной оплаты по цене ниже номинальной стоимости, и банк возвращает вам полную сумму счета до истечения срока оплаты. датировать. Например, счет за электроэнергию для XYZ составляет 1000 долларов США; компания, выставляющая счета за электроэнергию, продаст счет банку со скидкой от 10% до 20% к номинальной стоимости. Здесь банк купит счет за электроэнергию за 900 долларов США, номинальная стоимость которого составляет 1000 долларов США, теперь банк возместит полную сумму счета от клиента, то есть 1000 долларов США. Если клиент не оплачивает счет, банк начисляет проценты на непогашенный счет и требует от клиента оплаты.


    51) Что такое «Покупка счетов»?

    В «Покупке векселей» ссуда будет создана на полную стоимость тратты, а проценты будут возмещены, когда придет фактическая оплата.

График y cos x 2: Построить график функции y=cos x/2

y = cos(x/2)

Графики функций, Построение графиков Работа проверена: zcxfcnkbdfz Время решения: 4 мин Сложность: 4.6

Дано

$$f{left (x right )} = cos{left (frac{x}{2} right )}$$

График функции

Точки пересечения с осью координат X

График функции пересекает ось X при f = 0
значит надо решить уравнение:
$$cos{left (frac{x}{2} right )} = 0$$
Решаем это уравнение
Точки пересечения с осью X:

Аналитическое решение
$$x_{1} = pi$$
$$x_{2} = 3 pi$$
Численное решение
$$x_{1} = 9.42477796077$$
$$x_{2} = 84.8230016469$$
$$x_{3} = -53.407075111$$
$$x_{4} = 65.9734457254$$
$$x_{5} = 3.14159265359$$
$$x_{6} = 15.7079632679$$
$$x_{7} = -3.14159265359$$
$$x_{8} = 40.8407044967$$
$$x_{9} = -59.6902604182$$
$$x_{10} = 97.3893722613$$
$$x_{11} = 78. 5398163397$$
$$x_{12} = -34.5575191895$$
$$x_{13} = 28.2743338823$$
$$x_{14} = 7517042.68028$$
$$x_{15} = -91.1061869541$$
$$x_{16} = 72.2566310326$$
$$x_{17} = -9.42477796077$$
$$x_{18} = -65.9734457254$$
$$x_{19} = -72.2566310326$$
$$x_{20} = 47.1238898038$$
$$x_{21} = -84.8230016469$$
$$x_{22} = -9591.28237141$$
$$x_{23} = 91.1061869541$$
$$x_{24} = 59.6902604182$$
$$x_{25} = -47.1238898038$$
$$x_{26} = -21.9911485751$$
$$x_{27} = -97.3893722613$$
$$x_{28} = 34.5575191895$$
$$x_{29} = 21.9911485751$$
$$x_{30} = -160.221225333$$
$$x_{31} = 53.407075111$$
$$x_{32} = -78.5398163397$$
$$x_{33} = -40.8407044967$$
$$x_{34} = -15.7079632679$$
$$x_{35} = -28.2743338823$$

Точки пересечения с осью координат Y

График пересекает ось Y, когда x равняется 0:
подставляем x = 0 в cos(x/2).
$$cos{left (frac{0}{2} right )}$$
Результат:
$$f{left (0 right )} = 1$$
Точка:

(0, 1)

Экстремумы функции

Для того, чтобы найти экстремумы, нужно решить уравнение
$$frac{d}{d x} f{left (x right )} = 0$$
(производная равна нулю),
и корни этого уравнения будут экстремумами данной функции:
$$frac{d}{d x} f{left (x right )} = $$
Решаем это уравнение
Корни этого ур-ния
$$x_{1} = 0$$
$$x_{2} = 2 pi$$
Зн. {2}} f{left (x right )} = $$
Решаем это уравнение
Корни этого ур-ния
$$x_{1} = pi$$
$$x_{2} = 3 pi$$

Интервалы выпуклости и вогнутости:
Найдём интервалы, где функция выпуклая или вогнутая, для этого посмотрим, как ведет себя функция в точках перегибов:
Вогнутая на промежутках

[pi, 3*pi]

Выпуклая на промежутках

(-oo, pi] U [3*pi, oo)

Горизонтальные асимптоты

Горизонтальные асимптоты найдём с помощью пределов данной функции при x->+oo и x->-oo
$$lim_{x to -infty} cos{left (frac{x}{2} right )} = langle -1, 1rangle$$
Возьмём предел
значит,
уравнение горизонтальной асимптоты справа:
$$y = langle -1, 1rangle$$

Наклонные асимптоты

Наклонную асимптоту можно найти, подсчитав предел функции cos(x/2), делённой на x при x->+oo и x ->-oo
$$lim_{x to -infty}left(frac{1}{x} cos{left (frac{x}{2} right )}right) = 0$$
Возьмём предел
значит,
наклонная совпадает с горизонтальной асимптотой слева

Чётность и нечётность функции

Проверим функци чётна или нечётна с помощью соотношений f = f(-x) и f = -f(-x).
Итак, проверяем:
$$cos{left (frac{x}{2} right )} = cos{left (frac{x}{2} right )}$$
– Нет
$$cos{left (frac{x}{2} right )} = – cos{left (frac{x}{2} right )}$$
– Нет
значит, функция
не является
ни чётной ни нечётной

   

тригонометрия — график $\sin(x) + \sin(y)= \cos(x) + \cos(y)$

спросил

Изменено 2 года, 5 месяцев назад

Просмотрено 428 раз

$\begingroup$

Я играл с неявными графиками вида $f(x,y) = g(x,y)$ и заметил, что если построить на плоскости следующее уравнение: $\sin(x) + \ sin(y)= \cos(x) + \cos(y)$ получаем следующий график:

Мой вопрос: почему эта тригонометрическая функция дает нам эти квадраты, охватывающие всю плоскость?

  • тригонометрия
  • графические функции
  • неявные функции

$\endgroup$

1

$\begingroup$

Использование формул простафаэреза

$$2\sin\dfrac{x+y}2\cos\dfrac{x-y}2=\cos\dfrac{x+y}2\cos\dfrac{x-y}2$$

Если $\cos\dfrac{x-y}2=0\имплицитно\dfrac{x-y}2=(2n+1)\dfrac\pi2, x-y=(2n+1)\pi$

иначе $\sin\ dfrac {x + y} 2 = \ cos \ dfrac {x + y} 2 \ iff \ tan \ dfrac {x + y} 2 = 1 \ подразумевает \ dfrac {x + y} 2 = m \ pi + \ dfrac \ pi4 \iff x+y=\dfrac{(4m+1)\pi}2$

Итак, мы получаем непрерывные перпендикулярные и равноудаленные прямые линии.

В первом случае расстояние между двумя последовательными линиями равно $$\dfrac{2(m+1)+1-(2m+1)}{\sqrt2}\cdot\pi$$

, а во втором , $$\dfrac{2\pi}{\sqrt2}$$

Итак, мы получаем бесконечное количество квадратов с каждой стороной $=\sqrt2\pi$

$\endgroup$

$\begingroup$

$$\sin(x)+\sin(y)=\cos(x)+\cos(y)\iff$$

$$\sin(x)-\cos(x)=\cos( y)-\sin(y)\iff$$

$$\sqrt{2}\sin(x-\frac{\pi}{4})=\sqrt{2}\sin(\frac{\pi {4}-y)\iff$$

$$x=-y+\frac{\pi}{2}+2k\pi$$ или же $$x=y+\pi+2k\pi$$ таким образом, есть два типа линий: возрастающие линии с уравнением $$y=x+(2k+1)\pi$$ и убывающие $$y=-x+(\frac 12+2k)\pi$$

где $ k\in \Bbb Z.$

$\endgroup$

$\begingroup$

\begin{выравнивание*} \sin(x)-\cos(x)&=\cos(y)-\sin(y)\стрелка вправо \sin\left(x-\frac{\pi}4\right)=\sin\left(\ frac{\pi}4-y\right)\\ &\Стрелка вправо x-\frac{\pi}4=2n\pi+\left(\frac{\pi}4-y\right)\\ &=x+y=2n\pi+\frac{\pi}2 \конец{выравнивание*} Повторить для $x-\frac{\pi}4=n\pi-\left(\frac{\pi}4-y\right)$

Семейство прямых 🙂

$\endgroup$

$\begingroup$

$$\sin(x)-\cos(x)=\cos(y)-\sin(y)$$ $$(\sin(x)-\cos(x))/\sqrt 2=(\cos(y)-\sin(y))/\sqrt 2$$

$$ \sin(x-\pi /4) = \sin (\pi/4-y) $$

Поясним для двух главных функций обратного синуса

$$x- \pi/4= \pi/4-y \rightarrow x+y = \ пи/2$$ $$x- \pi/4= \pi-[\pi/4-y] \rightarrow y=x-\pi$$

Вы видите эти две прямые линии на своем графике вокруг начала координат. Другой котерминальный обратный угол с периодами $2k\pi$.

$\endgroup$

Зарегистрируйтесь или войдите в систему

Зарегистрируйтесь с помощью Google

Зарегистрироваться через Facebook

Зарегистрируйтесь, используя электронную почту и пароль

Опубликовать как гость

Электронная почта

Требуется, но никогда не отображается

Опубликовать как гость

Электронная почта

Требуется, но не отображается

Нажимая «Опубликовать свой ответ», вы соглашаетесь с нашими условиями обслуживания, политикой конфиденциальности и политикой использования файлов cookie

.

График y= cos x для -3pi/2 меньше или равно x меньше или равно -pi/2 Какой самый большой

ПОЖАЛУЙСТА, МНЕ НУЖНА ПОМОЩЬ КАК МОЖНО СКОРЕЕ!!! ПОКАЖИТЕ МНЕ, КАК ВЫ ПОЛУЧИЛИ ОТВЕТ

Стоимость трейловой смеси в продуктовом магазине обычно составляет 12 долларов за фунт, но теперь цена увеличилась на 30%. Какова цена за фунт трейл ми … х теперь, когда цена увеличилась?

1. Числовая прямая показывает график неравенства: Числовая прямая показана от отрицательной 5 до положительной 5 с шагом 0,5. Все целые числа … отмечены на числовой прямой. На первой отметке слева от 0 показан пустой кружок. Область слева от пустого кружка заштрихована. Какое утверждение объясняет, может ли −2,5 быть значением в заштрихованной области? aДа, может, потому что −2,5 лежит левее −0,5. bДа, может, потому что −2,5 лежит правее −0,5. cНет, не может, потому что −2,5 лежит левее −0,5. dНет, не может, потому что −2,5 лежит правее −0,5,2. Если x = − 4, какая числовая строка показывает значение |x|? (5 баллов) aЧисловая линия показана от отрицательной 6 до положительной 6 с шагом 1. Все числа отмечены на числовой строке. В точке, находящейся в 4 делениях слева от 0, показана точка. Над числовой линией показана скобка, проходящая от точки в 4 делениях слева от нуля до 0. Между скобками показана горизонтальная линия. , а в строке написано 4. bЧисловая строка показана от отрицательной 6 до положительной 6 с шагом 1. Все числа отмечены на числовой строке. В точке, находящейся в 4 делениях слева от 0, показана точка. Над числовой линией показана скобка, проходящая от точки в 4 делениях слева от нуля до 0. Между скобками показана горизонтальная линия. , а в строке написано минус 4. cЧисловая строка показана от отрицательной 6 до положительной 6 с шагом 1. Все числа отмечены на числовой строке. Точка показана в точке на 4 деления слева от 0. Над числовой линией показана скобка, простирающаяся от точки на 4 деления слева от нуля до точки на 4 деления справа от нуль.

Задачи на нок: Веселый ранец — Текстовые задачи на нахождение НОД и НОК

НОК и НОД нескольких натуральных чисел

Тема: «НОК и НОД нескольких натуральных чисел»

№1. Найти НОК и НОД чисел:

а) 7 и 8; б) 2; 6 и 18; 3) 3; 9; 2; 4) 36 и 60; 5) 16 и 24

№2. Какую цифру надо записать вместо звездочки в числе 542*, чтобы оно:

а) делилось на 9; б) делилось на 5; в) делилось на 6?

№3. Задача. На спортивных соревнованиях участвовали равные по составу команды, всего 145 мальчиков и 87 девочек. Во всех командах было одинаковое количество мальчиков и одинаковое количество девочек. Сколько команд участвовало в соревнованиях? Сколько мальчиков и сколько девочек было в каждой команде?

№4. Задача. Какое наименьшее число метров материала должно быть в рулоне, чтобы его можно было продать по 8 метров или по 12 метров?

№5. Решить уравнения: а) (х+2,3)•0,2=0,7 б) 4,2х+8,4=14,7.

Дома: п.6, п.7, правила, №348, №426(а).

Найдите НОД и НОК чисел: 1) 32 и 96; 2) 32 и 15; 3) 20 и 36; 4) 14; 28 и 56;

5) 16; 36 и 40; 6) 35; 42 и 21; 7) 48; 18 и 54; 8) 72; 18 и 30.

Тема: «НОК и НОД нескольких натуральных чисел»

№1. Найти НОК и НОД чисел:

а) 7 и 8; б) 2; 6 и 18; 3) 3; 9; 2; 4) 36 и 60; 5) 16 и 24

№2. Какую цифру надо записать вместо звездочки в числе 542*, чтобы оно:

а) делилось на 9; б) делилось на 5; в) делилось на 6?

№3. Задача. На спортивных соревнованиях участвовали равные по составу команды, всего 145 мальчиков и 87 девочек. Во всех командах было одинаковое количество мальчиков и одинаковое количество девочек. Сколько команд участвовало в соревнованиях? Сколько мальчиков и сколько девочек было в каждой команде?

№4. Задача. Какое наименьшее число метров материала должно быть в рулоне, чтобы его можно было продать по 8 метров или по 12 метров?

№5. Решить уравнения: а) (х+2,3)•0,2=0,7 б) 4,2х+8,4=14,7.

Дома: п.6, п.7, правила, №348, №426(а).

Найдите НОД и НОК чисел: 1) 32 и 96; 2) 32 и 15; 3) 20 и 36; 4) 14; 28 и 56;

5) 16; 36 и 40; 6) 35; 42 и 21; 7) 48; 18 и 54; 8) 72; 18 и 30.

Тема: «НОК и НОД нескольких натуральных чисел»

№1. Найти НОК и НОД чисел:

а) 7 и 8; б) 2; 6 и 18; 3) 3; 9; 2; 4) 36 и 60; 5) 16 и 24

№2. Какую цифру надо записать вместо звездочки в числе 542*, чтобы оно:

а) делилось на 9; б) делилось на 5; в) делилось на 6?

№3. Задача. На спортивных соревнованиях участвовали равные по составу команды, всего 145 мальчиков и 87 девочек. Во всех командах было одинаковое количество мальчиков и одинаковое количество девочек. Сколько команд участвовало в соревнованиях? Сколько мальчиков и сколько девочек было в каждой команде?

№4. Задача. Какое наименьшее число метров материала должно быть в рулоне, чтобы его можно было продать по 8 метров или по 12 метров?

№5. Решить уравнения: а) (х+2,3)•0,2=0,7 б) 4,2х+8,4=14,7.

Дома: п.6, п.7, правила, №348, №426(а).

Найдите НОД и НОК чисел: 1) 32 и 96; 2) 32 и 15; 3) 20 и 36; 4) 14; 28 и 56;

5) 16; 36 и 40; 6) 35; 42 и 21; 7) 48; 18 и 54; 8) 72; 18 и 30.

Адрес публикации: https://www.prodlenka.org/metodicheskie-razrabotki/224485-reshenie-zadach-po-teme-nok-i-nod-neskolkih-n

НОД НОК — Задачи — Eolymp

Отправленные решения: 2493
Засчитанные решения: 987
Количество пользователей отправивших решение: 849
Количество пользователей решивших задачу: 715
Сложность: 16%
Сложность задачи определяется по соотношению количества пользователей, правильно решивших задачу, к количеству пользователей, отправивших решение по задаче.

#ПользовательВремя отправкиВремяПамятьЯзык
12038420Gungnir 3 нояб. 2022 г., 22:43:30 1 ms 76 KB C++ 17 (gnu 10.2)
11688799FB02_Sapehin 6 окт. 2022 г., 22:15:10 1 ms 76 KB C++ 17 (gnu 10.2)
12059316artsabko 6 нояб. 2022 г., 13:32:32 1 ms 76 KB C++ 17 (gnu 10.2)
12265956Rufatmusayev 23 нояб. 2022 г., 17:44:40 1 ms 76 KB C++ 17 (gnu 10.2)
6335872Bleefmer 12 дек. 2019 г., 14:27:45 1 ms 200 KB Pascal (fpc 3.2)
6335893Cocacool 12 дек. 2019 г., 14:29:06 1 ms 200 KB Pascal (fpc 3.2)
10428270p1ckerzwer 4 февр. 2022 г., 10:30:19 1 ms 456 KB Pascal (fpc 3.2)
8331426vas_g 25 янв. 2021 г., 19:47:07 2 ms 456 KB Pascal (fpc 3.2)
9210577vjudge4 22 авг. 2021 г., 05:39:37 1 ms 528 KB C++ 11 (gnu 10.2)
8258018vjudge3 14 янв. 2021 г., 07:50:16 1 ms 536 KB C++ 11 (gnu 10.2)
9991426NursutanU 29 нояб. 2021 г., 05:18:53 1 ms 580 KB C++ 17 (gnu 10.2)
10932223hasanov.r 23 апр. 2022 г., 15:39:36 1 ms 580 KB C++ 17 (gnu 10.2)
11762474Namchuk_Maksym_Fi-02 12 окт. 2022 г., 18:02:14 1 ms 580 KB C++ 11 (gnu 10.2)
11782187kowalski 14 окт. 2022 г., 11:11:06 1 ms 580 KB C++ 17 (gnu 10.2)
12301051Fatime42elizade 27 нояб. 2022 г., 14:57:39 1 ms 580 KB C++ 11 (gnu 10.2)
12347645zeynebhesenova 2 дек. 2022 г., 11:52:17 1 ms 580 KB C++ 11 (gnu 10.2)
12402779Turab111 8 дек. 2022 г., 17:12:58 1 ms 580 KB C++ 17 (gnu 10.2)
12410346leshrza 9 дек. 2022 г., 16:14:52 1 ms 580 KB C++ 17 (gnu 10.2)
12423357omar.huseynov 11 дек. 2022 г., 10:38:27 1 ms 580 KB C++ 11 (gnu 10.2)
12438471sefiqe.memisova 12 дек. 2022 г., 17:30:52 1 ms 580 KB C++ 17 (gnu 10.2)
12483034TuranEldarov 17 дек. 2022 г., 17:00:11 1 ms 580 KB C++ 17 (gnu 10.2)
11270405vjudge2 16 июл. 2022 г., 12:43:53 1 ms 584 KB C++ 17 (gnu 10.2)
11270522vjudge5 16 июл. 2022 г., 13:26:58 1 ms 584 KB C++ 17 (gnu 10.2)
10916397Elcan2006 20 апр. 2022 г., 14:13:18 1 ms 584 KB C++ 17 (gnu 10.2)
12389742memmedlihemid 6 дек. 2022 г., 21:49:47 1 ms 584 KB C++ 17 (gnu 10.2)

NOK CORPORATION

Новости/события

    • ИК

    Уведомление о дивидендах (промежуточных дивидендах)

    • ИК

    Консолидированные финансовые результаты за шесть месяцев, закончившихся 30 сентября 2022 г. (ОПБУ Японии)

    • ИК

    Уведомление о пересмотре прогноза (188 КБ)

    • ПРЕСС-РЕЛИЗ

    NOK инвестирует в фонд UMI III. Первая инвестиция в венчурный фонд; ускорение создания нового бизнеса (368 КБ)

    • ПРЕСС-РЕЛИЗ

    Уведомление об организационных и кадровых изменениях NOK (232 КБ)

Список новостей

Список событий

Руководство по применению

  • Автомобильное промышленное оборудование

    Продукция NOK Group широко используется в автомобилях. Мы поддерживаем безопасную и комфортную жизнь с помощью наших высокофункциональных и экологически чистых продуктов.

    Читать больше

  • Общее промышленное оборудование

    Различные продукты, созданные с помощью основных технологий NOK, используются в самых разных областях, включая строительную технику, сельскохозяйственную технику, жилищное оборудование, заводы и роботов.

    Читать больше

  • Информационное электронное оборудование

    Эти продукты поддерживают удобный образ жизни благодаря применению в различных электронных устройствах. Они варьируются от смартфонов, которые сейчас необходимы в нашей жизни, до планшетов, носимых устройств и автомобилей следующего поколения.

    Читать больше

  • Оборудование для автоматизации офиса

    Ролики широко используются вокруг фоторецепторов, определяющих производительность копировальных аппаратов, и для фиксации деталей, требующих высокой функциональности и долговечности.

    Читать больше

Специальный

Группа норвежских крон

    • ОРЕЛ ИНДУСТРИИ КО., ЛТД.
    • НИППОН МЕКТРОН, ЛТД.
    • СИНЗТЕК КО., ЛТД.
    • НОК КЛЮБЕР КО., ЛТД.
    • ЮНИМАТЕК КО., ЛТД.
    • Общее товарищество Freudenberg-NOK
    • Thai NOK Co., Ltd.
    • ПТ. NOK Индонезия
    • Вьетнам NOK Co., Ltd.
    • Wuxi NOK-Freudenberg Oilseal Co., Ltd.
    • Чанчунь NOK-Freudenberg сальник Co., Ltd.
    • NOK-Freudenberg Group Sales (China) Co., Ltd.
    • NOK-Freudenberg Group Trading (China) Co., Ltd.
    • Фройденберг NOK Pvt. , Ltd.
    • Pyung Hwa Oil Seal Industry Co., Ltd.
    • NOK Европа ГмбХ

БОЛЕЕ

Ближайшие родственники Значение в Северной Каролине

Когда кто-то умирает, его ближайшие родственники (NOK) обычно уведомляются первыми. Они также принимают решения о том, что будет дальше с останками умершего человека. Это время потери может быть тяжелым для них, так как теперь они должны оплакивать смерть близкого человека. В Северной Каролине NOK представляет собой конкретный список людей в жизни умершего человека, наделенных полномочиями принимать решения относительно его останков. Давайте подробнее рассмотрим значение ближайших родственников и решения, которые они должны принять, когда умирает любимый человек.

Что делает ближайший родственник (NOK)?

Когда кто-то умирает в Северной Каролине, его ближайшие родственники должны принять несколько важных решений. К ним относятся выбор организации похорон и погребения, решение последних дел умершего и уведомление соответствующих властей. Ближайшие родственники могут делегировать некоторые из этих задач друзьям или специалистам.

Эти решения могут быть сложными для человека, особенно если он скорбит. Это нормально и ожидается, что ближайшие родственники делегируют некоторые из этих задач друзьям или профессионалам, например, местное похоронное бюро и крематорий.

Однако, если вы не хотите, чтобы ваши норвежские кроны были обременены этими непреодолимыми решениями, важно спланировать похороны заранее.

Кто является ближайшим родственником (NOK) в Северной Каролине?

Согласно Закону Северной Каролины NCGSA § 130A-420, существует порядок, в котором вы можете найти ближайших родственников (NOK). Именно они будут принимать решения об останках умершего человека и его имуществе в целом.

Ниже приведен порядок, которым должны руководствоваться похоронные бюро при поиске человека, который может дать инструкции по захоронению останков. Давайте рассмотрим эти роли в определенном порядке закона Северной Каролины и посмотрим, как каждая из них работает по очереди.

Номер 1: Официально назначенные роли и документы

Многие люди планируют свою смерть, назначая кого-то с полномочиями принимать решения за них после их смерти. Есть много способов назначить кого-то, кому вы доверяете, для принятия важных решений о ваших останках после вашей смерти. Вы также можете использовать юридические документы для изложения своих пожеланий, если они юридически приемлемы в нашем штате.

Вы можете назначить любого человека старше 18 лет или использовать документ, чтобы изложить свои пожелания с помощью этих инструментов планирования законного имущества:

Доверенность на медицинское обслуживание:

Этот юридический документ свидетельствует о том, что указанное лицо имеет полномочия принимать за вас все медицинские или медицинские решения или распоряжаться вашими останками после вашей смерти.

Предварительное соглашение:

Это юридическое соглашение между вами и похоронным бюро, в котором вы вносите предоплату или заключаете с ними договор на обработку деталей ваших останков.

Последняя воля и завещание:

В вашем завещании указан ваш душеприказчик, который принимает все решения относительно вашего имущества и договоренностей после вашей смерти.

Официальное заявление, подписанное двумя взрослыми свидетелями:

Это может быть рукописное завещание или другой документ, в котором умерший излагает планы относительно своего останка после смерти.

Другие письменные юридические заявления в соответствии с законодательством Северной Каролины:

Ваш адвокат может посоветовать вам различные юридические способы назначения лица для принятия решений после вашей смерти, например, договоры о доверительном управлении.

Подписанная форма DD 93 с указанием того, кому передается их тело после смерти:

Для военнослужащих, форма DD 93 определяет получателей определенных пособий в случае смерти военнослужащего. Это также руководство по распределению заработной платы и пособий этого члена в случае захвата, пропажи или интернирования с указанием имен и адресов людей, которых военнослужащий хочет уведомить в случае чрезвычайной ситуации или смерти. Гражданский персонал может использовать форму для ускорения процесса уведомления в случае чрезвычайной ситуации или собственной смерти.

Опекун, назначенный судом, может планировать смерть человека:

Опекун — это лицо, имеющее законные полномочия заботиться о другом человеке и принимать за него решения. Если существует назначенный судом опекун, он имеет право составить план на случай смерти человека до того, как он умрет . Однако, как только человек умирает, опекун больше не имеет права принимать решения, если только он не является NOK.

Номер 2: Оставшийся в живых супруг

Если отсутствуют юридические документы, подтверждающие доверенность на медицинское обслуживание умершего, право принятия решения принадлежит оставшемуся в живых супругу.

В этом случае супруг часто оказывается в отчаянии и пытается принять решение о том, в какое похоронное бюро позвонить, как справиться с этой потерей, заботиться о детях, задаваться вопросом, как выжить без дохода или роли супруга, и что делать с останками.

Быть NOK — тяжелое бремя для супруга. Поскольку супругам приходится справляться с такой большой нагрузкой, многие люди составляют документы по планированию имущества, чтобы их супруги знали об их желаниях и планах.

Оставив документы по планированию наследства и предварительный план похоронного бюро, в котором точно указано, чего вы хотите, дайте вашим близким больше покоя относительно того, что произойдет дальше.

Номер 3: Большинство взрослых детей 

Если нет доверенности на медицинское обслуживание и нет супруга/супруги, решения об останках принимаются большинством взрослых детей. Если дети недоступны или не могут принимать решения вместе, решения переходят к родителям.

Часто большинству взрослых детей приходится решать, как распоряжаться останками. Кремация является окончательной, поэтому иногда взрослые дети расходятся во мнениях по поводу кремации и захоронения.

Иногда попытка связаться со всеми детьми может отсрочить окончательное решение.

Однако, если похоронное бюро предприняло добросовестные усилия, чтобы связаться с детьми, но только один из них оказался доступным, они могут подписать формы и принять решение о захоронении или кремации.

Номер 4: Родители 

Очевидно, что большинство родителей проявляют заботу и заботу о своих детях, когда они умирают.

Номер 5: Большинство взрослых братьев и сестер 

Если ваши родители и дети недоступны, эта обязанность ложится на ваших братьев и сестер. Любые доступные братья и сестры обсуждают и решают, что лучше для следующих шагов.

Номер 6: Большинство лиц в следующих степенях родства

Сюда может входить большинство ваших двоюродных братьев или сестер или большинство ваших кровных дядей и тетей.

Номер 7: Кто-то, кто проявлял к вам особую заботу и заботу

Этот человек также должен быть готов и способен принимать решения относительно вашего тела. Возможно, у вас был друг, который всегда приходил навестить вас в доме престарелых, и персонал знал его как заботливого человека в вашей жизни.

Номер 8: Государственный служащий

Иногда представитель социальных служб, проводящий проверку состояния здоровья, или сотрудник правоохранительных органов после сообщения об обнаружении кого-то умершим дома. Если у человека нет планирования имущества или информации о каких-либо близких, друзьях или семье, государственное должностное лицо может принять решение об их останках.

Номер 9: Представитель учреждения:

Когда человек преклонного возраста не планирует свою смерть, он часто умирает в доме престарелых или в доме престарелых. Если у них нет семьи или друзей, решение об их останках может принимать глава дома престарелых или престарелых.

Номер 10: Любое другое лицо, желающее взять на себя ответственность

Если ни одно из вышеперечисленных лиц не возьмет на себя ответственность и не будет документов по планированию недвижимости, любой юридический или юрисдикционный орган, желающий взять на себя процесс принятия решения, может Сделай так. Однако они также должны оплатить счет.

Кто НЕ считается ближайшим родственником?

Некоторые люди не являются ближайшими родственниками (если только первые десять норвежских крон недоступны). Эмоциональная или физическая близость к умершему не означает, что он становится NOK.

К ним относятся:

  • Соседи по комнате
  • Смотрители
  • Коллеги
  • Неженатые партнеры

Если вы хотите, чтобы кто-то, кроме ваших ближайших родственников, принимал решения о ваших останках, вам необходимо заранее спланировать это, используя документы планирования имущества. Вы можете назначить кого-то своим душеприказчиком и иметь юридические полномочия принимать решения от вашего имени.

Планирование своей смерти

Никто не любит думать о собственной смерти, но очень важно планировать будущее. Если вы не хотите, чтобы ваши ближайшие родственники принимали решения относительно ваших останков, вам необходимо назначить кого-то в письменной форме.

Вы можете назначить кого-то на основании доверенности на медицинское обслуживание или иным образом. Кроме того, подумайте о том, чтобы заранее составить план похорон, чтобы вашим близким не пришлось принимать решения о ваших окончательных приготовлениях во время скорби. Не помещайте свои планы похорон в свое завещание, так как обычно никто не просматривает его до нескольких дней или недель после смерти.

Принятие решений о том, что произойдет после вашей смерти, является неотъемлемой частью жизни. Планируя заранее, вы можете снять бремя со своих близких в трудное время.

Мы можем помочь

В похоронном бюро и крематории «Ренессанс» мы понимаем важность планирования вашей смерти.

Преобразование иррациональных выражений онлайн калькулятор: Упрощение выражений · Калькулятор Онлайн

Непрерывные, цепные дроби онлайн

  • Египетские дроби. Часть вторая
  • Египетские (аликвотные) дроби
  • По сегменту определить радиус окружности
  • Круг и площадь, отсекаемая перпендикулярами
  • Деление треугольника на равные площади параллельными
  • Определение основных параметров целого числа
  • Свойства обратных тригонометрических функций
  • Разделить шар на равные объемы параллельными плоскостями
  • Взаимосвязь между организмами с различными типами обмена веществ
  • Аутотрофные и миксотрофные организмы
  • Рассечение круга прямыми на равные площади
  • Период нечетной дроби онлайн. Первые полторы тысяч разложений.
  • Представить дробь, как сумму её множителей
  • Решение системы из двух однородных диофантовых уравнений
  • Расчет основных параметров четырехполюсника
  • Цепочка остатков от деления в кольце целого числа
  • Система счисления на базе ряда Фибоначчи онлайн
  • Уравнение пятой степени. Частное решение.
  • Рассчитать площадь треугольника по трем сторонам онлайн
  • Общее решение линейного диофантового неоднородного уравнения
  • Частное решение диофантового уравнения с несколькими неизвестными
  • Онлайн разложение дробно рациональной функции
  • Корни характеристического уравнения
  • Имя пользователя при работе с Excel
  • Распределение частот появления букв русского алфавита в текстах
Выражение или значение для которого рассчитывается цепная дробь
Рассчитываем элементы непрерывной дроби для числа
Полученный результат.Числитель
Знаменатель
Непрерывная дробь

Цепная или непрерывная дробь — это дробь которую можно отобразить в виде

\(m/n=a0+\cfrac{1}{a1+\cfrac{1}{a2+\cfrac{1}{a3+ . ..}}}\)

Цепочку из элементов a0,a1,a2,a3 и т.д. чаще всего представляют в виде строки [a0,a1,a2,a3,a4…..an]

Как пример  покажем как можно представить дробь  52/111 в виде цепной дроби

\(52/111=0+\cfrac{1}{2+\cfrac{1}{7+\cfrac{1}{2+\cfrac{1}{3}}}}\)

Применение цепных дробей достаточно велико.

Мы можем с помощью них находить приближенные значения иррациональных чисел.

Непрерывная дробь иррационального квадратного корня всегда имеет вид

Таким образом мы можем утверждать что цепная дробь периодическая.

Мы можем сокращать дроби и в идеальном случае создавать калькулятор дробных чисел, не обращая внимания на то, какие числа в выражении будут фигурировать.

Для любознательных и юных математиков, покажется очень интересным материал позволяющим по элементам непрерывной, цепной дроби (в том числе и комплексной) получать результирующую  дробь(Результат по комплексной цепной дроби). Анализ таких цепей при разных начальных условиях, дает широкую возможность оценить свои силы и возможности понимания этой части такой науки как арифметика.

 

Для тех пользователей кто попал сюда в поисках вычисления дробных выражений то Вам вот на этот ресурс Калькулятор правильных и неправильных дробей

12*(12/17+11/13) -2.4

 

Пишем это выражение как есть и получаем результат 


Рассчитываем элементы непрерывной дроби для числа
Полученный результат.Числитель

17928

Знаменатель

1105

Непрерывная дробь

16 4 2 5 7 2 1

 

 

Пример3

Попробуем сократить  заданную дробь 62345346 / 786786. Хотелось бы заметить что это так же задачей по нахождению наибольшего общего делителя.

 

Пишем 62345346/786786

 

ответ

 

Рассчитываем элементы непрерывной дроби для числа
Полученный результат. Числитель

1484413

Знаменатель

18733

Непрерывная дробь

79 4 6 2 1 4 2 1 3 4

 

  • НОД двух многочленов. Greatest Common Factor (GCF) >>
Поиск по сайту
  • Русский и английский алфавит в одну строку
  • Часовая и минутная стрелка онлайн.Угол между ними.
  • Универсальный калькулятор комплексных чисел онлайн
  • Массовая доля химического вещества онлайн
  • Перемешать буквы в тексте онлайн
  • Декoдировать текст \u0xxx онлайн
  • Частотный анализ текста онлайн
  • Поворот точек на произвольный угол онлайн
  • Обратный и дополнительный код числа онлайн
  • Площадь многоугольника по координатам онлайн
  • Остаток числа в степени по модулю
  • Расчет процентов онлайн
  • Как перевести градусы в минуты и секунды
  • Поиск объекта по географическим координатам
  • Расчет пропорций и соотношений
  • Время восхода и захода Солнца и Луны для местности
  • DameWare Mini Control. Настройка.
  • Растворимость металлов в различных жидкостях
  • Калькулятор географических координат
  • Расчет значения функции Эйлера
  • Перевод числа в код Грея и обратно
  • Теория графов. Матрица смежности онлайн
  • Географические координаты любых городов мира
  • Произвольный треугольник по заданным параметрам
  • Онлайн определение эквивалентного сопротивления
  • НОД двух многочленов. Greatest Common Factor (GCF)
  • Площадь пересечения окружностей на плоскости
  • Непрерывные, цепные дроби онлайн
  • Калькулятор онлайн расчета количества рабочих дней
  • Расчет заряда и разряда конденсатора через сопротивление
  • Построить ненаправленный граф по матрице
  • Сообщество животных. Кто как называется?
  • Месторождения золота и его спутники
  • Расчет понижающего конденсатора
  • Из показательной в алгебраическую. Подробно
  • Проекция точки на плоскость онлайн
  • Система комплексных линейных уравнений
  • Дата выхода на работу из отпуска, декрета онлайн
  • Определение формулы касательной к окружности
  • Каноническое уравнение гиперболы по двум точкам
Онлайн расчеты
Подписаться письмом

Математический помощник math-helper.ru

Видеоуроки математики / Элементарная математика

Координатная плоскость. Математика 6 класс. Урок №25 + видео

Координатная плоскость. Математика 6 класс. Урок №25 Взаимно перпендикулярные прямые Ох и Оу с выбранными на них направлениям и единичным отрезком образуют прямоугольную систему координат. Плоскость, на которой выбрана система координат, называется координатной плоскостью. Оси координат разделяют плоскость на четыре координатные четверти. Каждой точке на координатной плоскости соответствует пара чисел: …

Читать далее…

Видеоуроки математики / Элементарная математика

Решение уравнений. Математика 6 класс. Урок №24 + видео

Решение уравнений. Математика 6 класс. Урок №24 Равенство, которое содержит переменную, называется уравнением. Решение уравнения (корень) — это число, при подстановке которого в уравнение получают верное равенство. Решения уравнения не изменятся, если обе части уравнения умножить или разделить на одно и то же число, не равна нулю. Решения уравнения не …

Читать далее…

Видеоуроки математики / Элементарная математика

Деление рациональных чисел. Математика 6 класс. Урок №23 + видео

Деление рациональных чисел. Математика 6 класс. Урок №23 Деление — это действие, в которой по данным произведением и одним из множителей находят второй множитель. Разделить число на число — значит найти такое число , чтобы . Частное двух чисел с одинаковыми знаками — число положительное. Частное двух чисел с разными …

Читать далее…

Видеоуроки математики / Элементарная математика

Умножение рациональных чисел. Квадрат и куб числа. Урок №22 + видео

Умножение рациональных чисел. Умножение чисел с разными знаками. Квадрат и куб числа. Урок №22 + видео Правила умножения Для того, чтобы перемножить два числа с одинаковыми знаками, надо умножить модули этих чисел и поставить перед полученным произведением знак «+». Произведением двух отрицательных чисел является числом положительным. Для того, чтобы перемножить …

Читать далее…

Видеоуроки математики / Элементарная математика

Вычитание рациональных чисел. Вычитание чисел с разными знаками. Урок №21 + видео

Вычитание рациональных чисел. Вычитание чисел с разными знаками. Урок №21 + видео Для того, чтобы из данного числа вычесть другое, надо к уменьшающегося прибавить число, противоположное вычитаемому. Выражение, содержащее знаки сложения и вычитания, можно рассматривать как сумму. Разность двух чисел положительна, если уменьшаемое больше вычитаемого. Разность двух чисел отрицательна, если …

Читать далее…

Видеоуроки математики / Элементарная математика

Сложение рациональных чисел. Урок №20 + видео

Сложение рациональных чисел. Урок №20 Любое число при сложении с положительным числом увеличивается, а при сложении с отрицательным числом — уменьшается. Для того, чтобы сложить два числа с одинаковыми знаками, надо сложить их модули и перед суммой поставить их общий знак. Для того, чтобы сложить два числа с разными знаками, …

Читать далее…

Рациональное число в виде дроби

Учеба Математика Алгебра

Этот онлайн-калькулятор записывает рациональное число в виде дроби (отношение двух целых чисел), используя формулу бесконечной геометрической прогрессии.

Когда вы начинаете изучать геометрические последовательности, вы можете столкнуться с задачей, сформулированной так:

Запишите рациональное число 0,58333… как отношение двух целых чисел.

Конечно, в этом примере нас просят преобразовать повторяет десятичное число до дроби. Действительно, для решения этой задачи требуется формула бесконечного геометрического ряда. Этот калькулятор использует эту формулу, чтобы узнать числитель и знаменатель для данного повторяющегося десятичного числа. Решение и формулы описаны под калькулятором.

Обратите внимание, что в приведенной выше задаче повторяющаяся десятичная дробь неформально представлена ​​многоточием (три точки…). На самом деле существует несколько условных обозначений для представления повторяющихся десятичных дробей, но ни одно из них не принято повсеместно. Например, в США обозначение представляет собой горизонтальную линию (винкулум) над повторяющимися цифрами, а в некоторых частях Европы обозначение заключается в заключении повторяющихся цифр в круглые скобки. Калькулятор поддерживает два способа ввода повторяющегося десятичного числа: 0,58333… и 0,58(3) 9.0009

Рациональное число в качестве соотношения двух целых чисел

Рациональное число

Отношение двух целых чисел

Повторяя десятичная десятка

к процитированию Wikipedia, 1 A . представление числа, цифры которого являются периодическими (повторяют свои значения через равные промежутки времени), а бесконечно повторяющаяся часть не равна нулю. Бесконечно повторяющаяся последовательность цифр называется 9.0019 повторить или повторить . Если повторение равно нулю, это десятичное представление называется конечным десятичным числом, а не повторяющимся десятичным числом. Можно показать, что число рационально тогда и только тогда, когда его десятичное представление повторяется или заканчивается (т. е. имеет конечное количество цифр или начинает повторять конечную последовательность цифр). А рациональное число, по определению, — это любое число, которое может быть выражено как частное или дробь p/q двух целых чисел, числителя p и ненулевого знаменателя q.

Если у нас есть завершающая десятичная дробь, мы можем использовать конвертер дробей в десятичную и десятичную дробь. В случае повторяющегося десятичного числа расчет становится немного сложнее. И здесь нам помогут геометрические последовательности. Давайте воспользуемся приведенным выше примером и преобразуем рациональное число (мы знаем, что оно рационально, потому что его десятичное представление повторяется) 0,58333… в дробь, используя наши знания о геометрических последовательностях.

Представим наше рациональное число так:

Числа и т. д. можно рассматривать как члены геометрической последовательности, где первый член равен 0,003, а знаменатель равен 0,1.

Действительно, согласно формуле для n-го члена геометрической прогрессии: , имеем

Обратите внимание, что это члены бесконечного геометрического ряда, который сходится, так как модуль знаменателя меньше единицы. Формула суммы для сходящегося бесконечного ряда:

Таким образом, для нашей задачи имеем

И, наконец,

Мы можем складывать и затем упрощать, зная, что наименьшее общее кратное 50 и 300 равно 300, а наибольший общий делитель 175 и 300 равен 25


  1. Википедия: повторяющееся десятичное число ↩

URL-адрес скопирован в буфер обмена

Похожие калькуляторы
  • • Сумма частных сумм геометрического ряда
  • • Калькулятор геометрического ряда и решение задач
  • • Геометрическая прогрессия
  • • Сумма частичных сумм арифметической последовательности
  • • Калькулятор арифметической последовательности и средство решения задач
  • • Раздел алгебры ( 109 калькуляторов )

 #рациональная #последовательность

PLANETCALC, Рациональное число как дробь0001

Вы можете использовать этот калькулятор преобразования десятичной дроби в дробь, чтобы преобразовать повторяющуюся десятичную дробь в исходную дробную форму.

Просто введите повторяющуюся часть десятичной дроби (повторяющуюся) и ее неповторяющуюся часть (где применимо). Например, если вы конвертируете 0,6 в 2/3, оставьте неповторяющееся поле пустым.

Как преобразовать повторяющиеся десятичные дроби в дроби

Когда дробь представляется в виде десятичной дроби, она может принимать форму завершающей десятичной дроби; например:

3/5 = 0,6 и 1/8 = 0,125,

или повторяющееся десятичное число; например,

19/70 = 0,2714285 и 1/6 = 0,16

Полоска, изображенная выше, представлена ​​над повторяющимся элементом числовой строки. Это известно как повторение. Вы можете преобразовать дробь в десятичную, чтобы упростить сложение и вычитание величин. Однако в практической математике часто встречаются повторяющиеся десятичные дроби при преобразовании дробей в проценты или десятичные дроби, что снижает точность вычислений.

Вы можете преобразовать десятичную дробь в исходную дробь, выполнив шаги, описанные ниже. Однако, если вы хотите немного облегчить себе жизнь, воспользуйтесь нашим калькулятором преобразования десятичной дроби в дробную.

Шаг 1: Отделите неповторяющуюся часть десятичного числа от повторяющейся части. Например, предположим, что вы хотите преобразовать следующее число в дробь:

0,3210708

Полоса располагается над неповторяющейся частью десятичной дроби. Таким образом, вы должны отделить 321 от 0708.

Шаг 2: Запишите неповторяющуюся часть десятичной дроби в степени 10, содержащую столько нулей, сколько чисел содержится в неповторяющейся части десятичной дроби (включая все нули). Например, поскольку 321 состоит из трех чисел, мы представляем дробь как 321/1000.

Шаг 3: Запишите повтор с таким количеством девяток, сколько цифр в этом повторе (опять же, включая нули). Например, поскольку 0708 состоит из четырех цифр, он представляется как 0708/9.999. Затем разделите эту дробь на степень 10, примененную на шаге 2. Например, поскольку мы применили 1000 на шаге два, мы вычислим следующее: (0708/9999)/1000 = 0708/9999000 = 708/9999000.

Шаг 4: Сложите две дроби, полученные на шагах 2 и 3 соответственно (согласно правилам сложения дробей, убедитесь, что вы даете им общий знаменатель). Например:

321/1000 + 708/9999000

= 3209679/9999000 + 708/9999000

= 3210387/9999000

Шаг 5: Уменьшите дробь, полученную на шаге 4. Например, и 3210387, и 9999000 можно разделить на 3. Таким образом, мы делим числитель и знаменатель на 3, чтобы получить следующее:

1070129/3333000.

Это дробная часть, эквивалентная 0,3210708.

Почему этот метод работает?

С помощью алгебры можно продемонстрировать, что все повторяющиеся десятичные дроби являются рациональными числами. Например, допустим, у нас есть x = 0,3210708 . Следующие алгебраические шаги можно применить, чтобы продемонстрировать, что x может быть представлен в виде фракции:

x = 0,3210708

x = 321/1000 + 0,0000708

x — 321/1000 = 0,0000708

1000 9000 (x — 321/1000) = 0,0708

1000 (X — 321/1000).

Как написать корень в калькуляторе: Как пишется корень в онлайн калькуляторе?

Сочетания клавиш в Графическом калькуляторе на Mac

Поиск по этому руководству

В приложении «Графический калькулятор»  на Mac можно быстро выполнять многие действия, используя сочетания клавиш. Сочетания клавиш приведены ниже и в меню приложения «Графический калькулятор» в строке меню. В меню приложения сочетания клавиш представлены с помощью символов.

Примечание. Сочетания клавиш в приложениях могут зависеть от языка и раскладки клавиатуры, которые используются на Mac. Если приведенные ниже сочетания клавиш работают не так, как ожидалось, посмотрите правильные сочетания в меню приложения в строке меню. Текущую раскладку клавиатуры, также называемую источником ввода, можно посмотреть в панели «Клавиатура».

Кроме того, чтобы вставить специальный оператор или символ, можно выбрать «Окно» > «Показать окно «Палитра уравнений»» или воспользоваться всплывающим меню добавления элементов  справа от поля «Уравнение».

Индекс или подстрочный текст (k0)

_ (символ подчеркивания)

Квадратный корень (√2)

Option-V

Корень (³√27)

Shift-Option-V

Сумма ()

Option-W

Произведение ( ∏ )

Shift-Option-P

Интеграл ( ∫ )

Option-B

Pi (π)

Option-P

Советы по работе с уравнениями

Задача

Советы

Ввод уравнений и функций

Вводите их так же, как обычно пишете.

Например, x = 2y или f(t) = 1 + 3t.

Ввод константы

Используйте = или :=

Например, k = 1 или k :=1.

Ввод нескольких значений для константы

Заключите значения в фигурные скобки.

Например, k = { 1, 2, 3 }.

Ввод диапазона значений

Используйте многоточие (…).

Например, k = { 1…10 } или k = { 0, 5, 10…15 }.

Объединение определений

Используйте запятую.

Например, x = 1 + cos(3k-y), k={ 1…5 }.

Трехмерные графики

Действие

Сочетание клавиш

Перемещение

Перетягивайте, удерживая клавишу Command

Вращение

Перетягивайте, удерживая клавишу Option

Увеличение и уменьшение масштаба

Перетягивайте, удерживая клавишу Shift

См. такжеСоздание графика и добавление уравнений с помощью Графического калькулятора на MacСтатья службы поддержки Apple: «Сочетания клавиш Mac»

Максимальное количество символов: 250

Не указывайте в комментарии личную информацию.

Максимальное количество символов: 250.

Благодарим вас за отзыв.

Делаем калькулятор-конвертер на базе React. Часть 9: Подключаем Redux и Router

Поговорим о важных и популярных сторонних библиотеках для React: Redux и Router. Зачем они нужны и как можно применить эти библиотеки на практике. Расскажу на примере калькулятора, который мы создавали в предыдущих частях цикла. 

Предыдыдущая часть: Калькулятор-конвертер на базе React. Часть 8: Конвертер валют и собственный парсер данных

Что такое Redux

Redux – это сторонняя библиотека для управления состояниями JavaScript-приложений. Вы уже знаете, как работают состояния компонентов в Реакте.  Так вот Redux позволяет эти состояния перенести из кучи разных элементов в единую систему контроля, находящуюся обособленно от остальных файлов проекта, но при этом всегда в зоне досягаемости. То есть состояние множества компонентов может содержаться в едином хранилище. Его можно обновлять из любого Реакт-компонента и отображать в любом Реакт-компоненте. 

Redux позволяет упростить запутанные структуры передачи данных от родительских компонентов к наследникам и от наследников обратно к родителям. 

Трудно сказать, насколько целесообразно использование Redux в нашем приложении. Об эту тему было сломано уже немало копий, и я не буду пересказывать статьи на тему того, как важно применять те или иные технологии только в тех ситуациях, когда они действительно нужны. Я лишь продемонстрирую базовый пример работы Redux, а вы уже сами решите, нужен ли он вам здесь. Либо вы просто обучитесь работе с Redux и сможете применять его в других приложениях. 

Устанавливаем Redux

Сначала нужно загрузить саму утилиту как npm-пакет и вместе с ней подтянуть все зависимые пакеты, чтобы полностью настроить базовый вариант хранилища состояний.  

  • Открываем терминал в директории с нашим проектом. 

  • Вводим команду npm install react-redux

  • Затем вводим npm install @reduxjs/toolkit

По окончании работы обеих команд можно переходить к настройке непосредственно в приложении.

Базовая настройка Redux

Для начала надо определиться с тем, как вообще будет использоваться Redux и для чего он нам нужен. Так как количество сценариев применения Redux ограничивается лишь фантазией программистов, то конкретные причины применения Redux никто назвать не сможет. 

В нашем случае Redux будет хранить в себе историю любых вычислений. Будь-то подсчеты в калькуляторе или какие-то операции, выполненные конвертером. То есть наше поле History превратится в общедоступное пространство, в которое можно перенести информацию из любого режима работы приложения, чтобы сохранить полученные значения.  

С этой информацией в голове переходим к созданию своего первого Redux-хранилища. 

Сначала создаем файл store.js и сохраняем его в нашем проекте. Это и будет JavaScript-документ, содержащий в себе состояния компонентов. 

В качестве содержимого файла store.js нужно указать базовую конфигурацию Redux с ссылками на сопутствующие компоненты и с объектом, хранящим в себе состояния компонентов.

  • Импортируем конфигурационный файл configureStore. 

    
    import { configureStore } from '@reduxjs/toolkit'
  • Затем импортируем функцию изменения состояния с помощью Redux (мы создадим ее чуть позже). 

    
    import historyReducer from './historySlice'
  • После этого экспортируем уже измененный объект configureStore во внешние пространства (чтобы к нему можно было обратиться из сторонних компонентов приложения). Внутри него укажем функцию-reducer, отвечающую за изменение Redux-состояния.  

    
    export default configureStore({ reducer: { history: historyReducer, }, })

Закончив с хранилищем, переходим к созданию метода для изменения состояния. Он будет содержаться в файле historySlice.js, который мы создадим в корневой директории проекта.

А вот его содержимое:

  • Импорт функции createSlice из пакета @reduxjs/toolkit:

    
    import { createSlice } from '@reduxjs/toolkit'
  • Создание переменной, содержащей в себе вызов функции createSlice с необходимыми данными: 

    
    export const historySlice = createSlice( )
  • В качестве аргумента для createSlice выступит объект со всей необходимой информацией и методами для управления состояниями:

    • Имя – name: history

    • Состояние по умолчанию – initialState: { value: [], }

    • Методы управления состоянием (функция-reducer). В нашем случае это функция updateHistory, принимающая аргументы из сторонних компонентов при вызове и обрабатывающая их внутри Redux.


updateHistory: (state, action) => 
  state.value.push(action.payload)
}

Объявив каждый элемент и описав метод управления состоянием, остается экспортировать это все во внешнюю среду, чтобы потом импортировать в программу.

  • Экспортируем метод. 

    
    export const { historyUpdate } = historySlice.actions
  • Экспортируем состояние. 

    
    export const historyState = state = state.history.value
  • Экспортируем метод reducer целиком. 

    
    export default historySlice.reducer

Готово. Можно переходить ко второй части. 

Подключаем Redux к нашему приложению

У нас есть внешнее состояние, но на текущий момент оно никак не используется в нашем приложении. Мы будем изменять его из двух компонентов нашей программы, а отображаться оно будет в общей части интерфейса, видимой при использовании и калькулятора, и конвертера. 

Для работы с Redux-состояниями в React-приложении, надо импортировать хранилище store и компонент Provider в корневой файл программы (в нашем случае это index.js).

Получится:


import store from './components/store'

import { Provider } from 'react-redux'

Provider – это родительский компонент-обертка, в который нужно поместить весь код приложения целиком. Ранее мы создавали ChakraProvider. Так что надо и его поместить внутрь Provider от Redux. Также важно не забыть передать store в качестве аргумента для Provider. 


<Provider store={store}> </Provider>

Для начала добавим наше внешнее состояние в компонент Converter, чтобы опробовать его в полной мере и понять, работает ли оно вообще. 

Импортируем весь список нужных нам элементов.

  • Хуки из библиотеки Redux:

    
    import { useSelector, useDispatch } from 'react-redux'
  • Методы и состояние из нашего файла historySlice:

    
    import { updateHistory, historyState } from '../historySlice

Далее создаем для них переменные внутри компонента Converter. Переменная history должна использовать хук useSelector на состоянии history, чтобы хранить его в компоненте, а переменная dispatch позволяет быстро обратиться к хуку useDispatch(). 

Дальше дело техники. Чтобы отобразить в интерфейсе состояние history, мы просто записываем соответствующую переменную в методе return компонента Converter. А чтобы менять его, создадим отдельную клавишу, активирующую метод dispatch. 

В нашем случае нужно передавать в Redux запрос на запуск метода updateHistory с аргументом result (так как мы планируем передавать значение вычисления в конвертере).  


<Button onClick={() => dispatch(updateHistory(result))}

Получится подобный интерфейс. Он показывает результат преобразования одной единицы измерения в другую и кнопку отправки этого значения в состояние history.

При нажатии на эту кнопку результат конвертации попадет в Redux-состояние history и отобразится выше. 

Проблема заключается в том, что мы не получаем преимуществ от Redux, так как history не отображается в других элементах. Нам нужно само состояние показать в интерфейсе на более высоком уровне, чтобы мы могли лицезреть его всегда, а не только при работе с конвертером. Ведь по изначальной задумке и функция изменения состояния должна работать из нескольких разных компонентов. 

Поэтому мы импортируем все те же данные из Redux и historySlice, но уже в компонент App. Там создадим переменную history, обращающуюся к хуку useSelector, и вернем ее в return.

Теперь можно добавить кнопку Add to history в любой компонент приложения, в том числе и в калькулятор. Работать она будет так же, как и любые другие аналогичные кнопки в приложении. 

Можно слегка модифицировать содержимое переменной history в компоненте App, чтобы элементы состояния отображались в отдельных кнопках.

Получится вот такой интерфейс. 

Также в нашем приложении Redux может упростить процесс реализации Drag & Drop.

Комьюнити теперь в Телеграм

Подпишитесь и будьте в курсе последних IT-новостей

Подписаться

Что такое Router

Router – это популярный инструмент для Реакт-программ и один из важнейших компонентов любого веб-приложения, позволяющий связать отдельные компоненты вашего проекта с конкретными адресами в браузере. 

Каждый компонент для браузера выглядит именно как отдельная страница, что позволяет оставлять ссылки на отдельные компоненты приложения. Например, в нашем случае вы можете создать ссылку сразу на калькулятор, а не на конвертер (или наоборот). Также это поможет сохранять состояние приложения при его перезагрузке. Сейчас при нажатии F5 мы автоматически переключаемся на калькулятор. 

Устанавливаем Router

Мы будем использовать React-Router. Есть и другие варианты роутинга, но этот показался мне наиболее простым. С ним легче всего разобраться, да и процесс установки укладывается в одну команду. 

По завершении работы команды переходим к настройки роутинга в приложении. 

Подключаем Router к нашему приложению

Подключается роутер довольно легко, так же, как и другие сторонние библиотеки. Нужно просто обернуть приложение в компонент, идущий в комплекте с React Router. 

Импортируем нужный компонент в index.js. 


import { BrowserRouter } from 'react-router-dom'

Теперь оборачиваем нашу программу в роутер. Нужно «положить» все, что идет уровнем ниже ChakraProvider, в BrowserRouter, как показано на скриншоте. 

После этого переходим к настройке роутинга в компоненте App.js. Так как это практически корень нашей программы, отсюда мы и будем перемещаться в разные части созданного нами инструмента.  

Импортируем сюда три компонента, отвечающих за базовую функциональность роутера. Это блок Routes, в котором будут содержаться все пути (ссылки на отдельные компоненты приложения), блок Route, хранящий в себе отдельный компонент для рендеринга, и Link – элемент, позволяющий переключаться между разными элементами страницы.


import { Routes, Route, Link } from 'react-router-dom'

Теперь пути нужно разместить в интерфейсе. Их можно спокойной прописать в методе return того компонента, который является корневым в вашем приложении. Все сразу они отображаться не будут, потому что роутинг работает как условный рендеринг, то есть отображаемый на странице контент будет зависеть от выбранного на текущий момент пути. 

  • Вместо переменной application, использованной ранее, создаем блок Router. 

    
    <Routes> </Routes>
  • Внутри блока указываем все нужные на пути, прописывая для них ссылку и компонент, который надо отобразить:

    • Путь до базового компонента (то есть того, что будет отображаться в интерфейсе программы по умолчанию). Так как у нас в программе нет главного экрана или его аналога, то мы по базовому пути будем выдавать калькулятор:

      
      <Route path="/" element={<Calculator />} />
    • Путь до конвертера:

      
      <Route path="converter" element={<Converter />} />
    • Путь до калькулятора:

      
      <Route path="calculator" element={<Calculator />} />

Осталось лишь сделать интерфейс для переключения между путями. У нас уже есть меню, отвечающее за работу условного рендеринга (то есть то, что позволяет выбирать значение переменной application внутри компонента App). Вот его и модифицируем, заменив кнопки, меняющие значение переменной application на элементы Link, а также URL в адресной строке браузера. 

  • Для калькулятора этот элемент будет выглядеть так: 

    
    <Link to="/calculator">Calculator</Link>
  • Для конвертера – так: 

    
    <Link to="/converter">Converter</Link

Вот как будет выглядеть приложение при запуске (обратите внимание на адресную строку).

И вот что будет, если переключиться на конвертер в меню.

Теперь при ручном вводе адреса /converter вы будете попадать в нужную часть программы, и это состояние приложения не будет сбрасываться при перезагрузке страницы. 

Вместо заключения

Мы изучили два важных элемента React-приложений, часто используемых в проектах различных масштабов. Даже если они и не нужны в таком мелком приложении, как калькулятор, то в других веб-сервисах они точно пригодятся.

Продолжение: Калькулятор-конвертер на базе React. Часть 10: Деплой

Как найти квадратный корень без калькулятора?

Система счисления — это система, определенная для различных чисел и способов их расположения. Существует много типов систем счисления, но в основном хорошо известны 4 типа. Это двоичные системы счисления, десятичные системы счисления, восьмеричные системы счисления и шестнадцатеричные системы счисления. Десятичная система счисления в основном используется в математике, она включает числа от 0 до 9. Над числами выполняется несколько операций, например, нахождение квадратов и квадратных корней чисел, давайте подробно узнаем о квадратных корнях чисел,

Квадратный корень

Квадратный корень из числа — это значение, которое при умножении само на себя дает исходное число. Например, квадратный корень из 9 равен 3, при умножении 3 на себя получается исходное число 9. Символ, обозначающий квадратный корень в математике, — √.

Этот символ (√) называется подкоренным, а число внутри подкоренного символа называется подкоренным. Число или значение внутри корневого символа может быть правильным квадратом или несовершенным квадратом. Например, 4 — правильный квадрат, а 3 — несовершенный квадрат. Таким образом, исходя из характера значения внутри корня, окончательный ответ или квадратный корень может быть натуральным числом десятичного числа.
Теперь давайте узнаем, как вычислять квадратный корень из разных чисел.

Квадратные корни без калькулятора

Как определено выше, квадратный корень числа — это значение, которое при умножении само на себя дает только исходное число. Есть три способа найти квадратный корень без калькулятора

Разложение на простые множители

Это длинный, но простой метод нахождения квадратного корня из любого числа. Простая факторизация включает в себя поиск множителей этого числа, а затем объединение общих чисел в пары по два. наконец, извлечение квадратных корней из простых множителей. Давайте посмотрим на пример этого,

Вопрос: найти квадратный корень из 484

Решение:

484=2 × 2 × 11 × 11

Итак, √484 = 1 × 1 × 11 × 2 11 =22

Метод «Угадай и проверь»  

Этот метод используется для получения приблизительного значения любого числа. Метод предположения экономит время, поскольку он дает приблизительный диапазон значений, между которыми существует корень. это более эффективно, когда число внутри корня является несовершенным числом. Давайте посмотрим на пример этого,

Вопрос: найдите квадратный корень из 20.

Решение:

Начните метод угадывания и проверки, отметив, что, поскольку √16 = 4 и √25 = 5, то √20 должно быть между 4 и 5 В качестве второго шага, чтобы приблизиться к реальному ответу, возьмем число от 4 до 5. Предположим, что оно равно 4,5. Давайте возьмем квадрат 4,5, который будет равен 20,25, что больше 20, поэтому корень должен быть меньше 4,5, давайте выберем 4,4, квадрат 4,4 равен 19..36. таким образом, наиболее приближенный и точный корень из 20 равен 4,4

Метод деления в длину  

Это очень простой способ получить квадратный корень из несовершенных квадратов. Метод длинного деления в основном предпочтительнее других методов, поскольку он дает точный ответ. Разберем этот алгоритм на примере

Вопрос: найдите квадратный корень из 627

Решение:

Шаг 1 Сгруппируйте числа попарно справа налево, оставив одну или две цифры слева (здесь это 6).

Шаг 2 Задумайте число, квадрат которого меньше первого числа (6), его 2, Итак, запишите его так: запишите результат под 6, а затем вычтите, как показано ниже,
           

Шаг 4  Умножьте частное на 3 и запишите его в скобках с пустой строкой рядом с ним, как показано ниже,
          

Шаг 5 Теперь найдите число, которое при умножении на сорок будет меньше 225. Давайте угадаем 5. Тогда            45×5=225, что меньше 227. Запишите его, как показано ниже:

Шаг 6 , затем повторяя шаг 4, умножьте коэффициент с 2 записи его в скобках с пустой линией рядом с ним, как показано ниже,

Шаг 7 Повторяющий шаг 5, узнайте число, которое при умножении на пятьсот будет меньше 2000. Давайте угадаем 5, тогда 505×5=2525, что больше 2000, давайте угадаем 4, тогда 504×3=1512. Так что напишите это, как показано ниже,
   

Квадратный корень из 627 с двумя десятичными знаками равен 25,03, что является точным значением.

Проблемы образцов

Вопрос 1: Найдите квадратный корень из 144

Решение:

144 = 2 × 2 × 2 × 2 × 3 × 3

SO √144 = √ (2 × 2 × 3 × 3

. × 2 × 2 × 2 × 3 × 3) = 2 × 2 × 3 = 12

Вопрос 2: Найдите квадратный корень из 169

Решение:

169=13 × 13

Итак, √144= √(13 × 13) = 13

Вопрос 3: Найдите квадратный корень из 6 методом «Угадай и проверь».

Решение:

Начните угадывать и проверять метод, отметив, что, поскольку √9 = 3 и √4 = 2, то √6 должно быть между 2 и 3. В качестве второго шага, чтобы приблизиться к реальному ответу возьмем число от 2 до 3. Предположим, что оно равно 2,5. Возьмем квадрат 2,5, который будет равен 6,25, что больше 6, поэтому корень должен быть меньше 2,5. Выберем 2,4, квадрат 2,4 равен 15,76. Таким образом, наиболее приближенный и точный корень из 6 равен 2,4·9.0011  

python — Как включить квадратный корень в базовый калькулятор

Итак, я написал базовый калькулятор на python:

 import math
импорт системы
num1 = float(input("Введите число"))
оператор = ввод("Введите оператор")
num2 = float(input("Введите второе число"))
если оператор == '+':
    напечатать (число1) + (число2)
Элиф оператор == '-':
    печать (число1 - число2)
Элиф оператор == '*':
    печать (число1 * число2)
Элиф оператор == '/':
    печать (число1/число2)
Элиф оператор == "sqrt":
        печать (math.sqrt (num1))
еще:
    print("Неизвестный оператор")
 

Я хочу сделать так, чтобы команда квадратного корня функционировала таким образом, чтобы при выборе num1 и оператора квадратного корня она пропускала остальные и выдавала результаты напрямую. Вместо того, чтобы также выписывать num2 .

  • питон
  • калькулятор

3

Поместите его в функцию, чтобы вы могли вернуть , прежде чем запрашивать num2 , если это уместно:

 импорт математики
деф расчет():
    num1 = float(input("Введите число"))
    оператор = ввод("Введите оператор")
    если оператор == "sqrt":
        печать (math.sqrt (num1))
        возвращаться
    num2 = float(input("Введите второе число"))
    если оператор == '+':
        напечатать (число1) + (число2)
    Элиф оператор == '-':
        печать (число1 - число2)
    Элиф оператор == '*':
        печать (число1 * число2)
    Элиф оператор == '/':
        печать (число1/число2)
вычислить()
 

Другие унарные операции можно просто добавить как elif s под sqrt .

Другим подходом может быть размещение операторов в словарях в соответствии с количеством операндов:

 import math
unary_ops = {
 "sqrt": math.

Mg h3po4 уравнение реакции: Допишите уравнения:Mg+H3PO4= Cu+H3PO4= Mg(OH)2+HCl= Cu(NO3)2+zn= Zn(NO3)2+Cu

Фосфорная кислота: получение и свойства

 

 

Строение молекулы и физические свойства

 

Фосфор в степени окисления +5 образует несколько кислот: орто-фосфорную H3PO4, мета-фосфорную HPO3, пиро-фосфорную H4P2O7.

Фосфорная кислота H3PO4 – это кислота средней силы, трехосновная, прочная и нелетучая. При обычных условиях фосфорная кислота – твердое вещество, хорошо растворимое в воде и гигроскопичное.

 

 

Валентность фосфора в фосфорной кислоте равна V.

При температуре выше +213 °C орто-фосфорная кислота переходит в пирофосфорную H4P2O7.

 

 

При взаимодействии высшего оксида фосфора с водой на холоде образуется метафосфорная кислота HPO3, представляющая собой прозрачную стекловидную массу.

 

 

 

Способы получения

 

Наибольшее практическое значение из фосфорных кислот имеет ортофосфорная кислота.

1. Получить орто-фосфорную кислоту можно взаимодействием оксида фосфора (V) с водой:

P2O5   +   3H2O    →    2H3PO4

2. Еще один способ получения фосфорной кислоты — вытеснение фосфорной кислоты из солей (фосфатов, гидрофосфатов и дигидрофосфатов) под действием более сильных кислот (серной, азотной, соляной и др.).

Промышленный способ получения фосфорной кислоты обработка фосфорита концентрированной серной кислотой:

Ca3(PO4)2(тв)    +  3H2SO4(конц)  →   2H3PO4   +   3CaSO4

3. Фосфорную кислоту также можно получить жестким окислением соединений фосфора в водном растворе в присутствии кислот.

Например, концентрированная азотная кислота окисляет фосфор до фосфорной кислоты:

5HNO3      +    P     →   H3PO4     +   5NO2↑    +    H2O

 

Химические свойства

 

Фосфорная кислота – это кислота средней силы (по второй и третьей ступени слабая).

1. Фосфорная кислота частично и ступенчато диссоциирует в водном растворе.

H3PO4  ⇄  H+ + H2PO4

H2PO4  ⇄  H+ + HPO42–

 HPO42– ⇄ H+ + PO43–

 

2. Фосфорная кислота реагирует с основными оксидами, основаниями, амфотерными оксидами  и амфотерными гидроксидами

Например, фосфорная кислота взаимодействует с оксидом магния:

2H3PO4    +   3MgO   →   Mg3(PO4)2   +   3H2O

 

Еще пример: при взаимодействии фосфорной кислоты с гидроксидом калия образуются фосфаты, гидрофосфаты или дигидрофосфаты:

H3PO4    +   КОН     →     KH2РО4  +   H2O

H3PO4    +   2КОН      →     К2НРО4  +   2H2O

H3PO4    +   3КОН     →    К3РО4  +   3H2O

 

3. Фосфорная кислота вытесняет более слабые кислоты из их солей (карбонатов, сульфидов и др.).  Также фосфорная кислота вступает в обменные реакции с солями.

Например, фосфорная кислота взаимодействует с гидрокарбонатом натрия:

Н3PO4   +   3NaHCO3   →   Na3PO4   +   CO2   +  3H2O

 

4. При нагревании H3PO4  до 200°С происходит отщепление от нее молекулы воды с образованием пирофосфорной кислоты H2P2O7:

2H3PO4   →  H2P2O7   +   H2O

 

5. Фосфорная кислота взаимодействует с металлами, которые расположены в ряду активности металлов до водорода. При этом образуются соль и водород.

Например, фосфорная кислота реагирует с магнием:

2H3PO4    +   3Mg   →    Mg3(PO4)2   +   3H2

 

Фосфорная кислота взаимодействует также с аммиаком с образованием солей аммония:

2H3PO4   +   3NH3    →    NH4H2PO4     +   (NH4)2HPO4

 

7. Качественная реакция на фосфат-ионы и фосфорную кислоту — взаимодействие с нитратом серебра. При этом образуется ярко-желтый осадок фосфата серебра:

 

Н3PO4   +    3AgNO3    →   Ag3PO4↓  +   3НNO3

 

 

Видеоопыт взаимодействия фосфата натрия и нитрата серебра в растворе  (качественная реакция на фосфат-ион) можно посмотреть здесь.

Понравилось это:

Нравится Загрузка…

Физические и химические явления. Закон сохранения массы веществ. Уравнения химических реакций. Типы химических реакций презентация, доклад

Слайд 1
Текст слайда:

Физические и химические явления. Закон сохранения массы веществ. Уравнения химических реакций. Типы химических реакций.(8 класс)

Титович Е.Н.


Слайд 2
Текст слайда:

Тема урока: Физические и химические явления, Химические реакции Цель урока: Познакомить с признаками физических и химических явлений, научить записывать уравнения простейших реакций.

Явления

Физические

Химические

Замерзание воды
Изгибание стеклянной трубки
Плавление парафина

Горение Бумаги
Скисание молока
Потемнение медного подсвечника


Слайд 3
Текст слайда:

Признаки явлений Физических и Химических


Слайд 4

Слайд 5

Слайд 6
Текст слайда:

Закон сохранения массы веществ,уравнения химических реакций

М. В. Ломоносов

Роберт Бойль

А.Л. Лавуазье

А.


Слайд 7

Слайд 8
Текст слайда:

Уравнения химических реакций

Химическое уравнение- условная запись химической реакции с помощью химических формул и коэффициентов

С+О2=СО2

Р+ О2=

AL+ O2 =

Mg+CL2=

AL+S=

P O

+5

-2

2

5

5

2

4

Al O

-2

2

2

3

+3

4

3


Слайд 9
Текст слайда:

Запишите уравнения реакций, подтвердите расчётом закон сохранения массы веществ

Ca+ O2=
Mg+ N2=
AL+ CL2=
h3O= h3 +…
AL + HCl = AlCL3 + …
Ag2O= Ag + …
Na+ Cl2 =
K+ S =
CO+ Fe2O3 = CO2+ …


Слайд 10
Текст слайда:

Типы химических реакций

Обмена

Разложения

Замещения

А+В=АВ

АВ=А+В

А+ВС=АС+В

АВ+СД=СВ+АД

Соединения


Слайд 11

Слайд 12

Слайд 13
Текст слайда:

Закончить уравнения реакций и определить их тип

K+ N2=
AL+ Fe3O4= … + …
Ba+ O2=
KOH+ ALCL3= AL(OH)3+….
HgO= O2 + …
SO2+ O2= SO3
Na2O+ h3O=
Mg+ h4PO4= Mg3(PO4)2 +…

Для первого и второго уравнения подтвердить расчётом
закон сохранения массы веществ


Слайд 14
Текст слайда:

Запишите уравнения реакций между:

1 серной кислотой и гидроксидом алюминия
2 оксидом кальция и ортофосфорной кислотой
3 сульфатом алюминия и гидроксидом натрия
4 сульфатом меди(II) и алюминием
5 карбонатом кальция и азотной кислотой
6 нитратом железа(III) и гидроксидом калия
7 оксидом железа(III) и алюминием
8 алюминием и соляной кислотой
9 хлоридом бария и сульфатом натрия
10 сернистой кислотой и гидроксидом калия


Слайд 15
Текст слайда:

Самостоятельная работа

АL+O2=
Mg+ HCL=MgCL2+…
K2O+HCL= KCL+…
Ag2O=Ag+…

Na+O2=
BaO+HCL=BaCL2+…
h3O=h3+…
C+CuO=CO2+…

В-1

В-2

Для последнего уравнения
подтвердите расчётом закон
сохранения массы веществ.

Закончите уравнения реакций, расставьте
коэффициенты, укажите тип реакции.


Скачать презентацию

Видео с вопросами: расчет молярной концентрации Mg(OH)₂ с использованием данных эксперимента по титрованию

Концентрацию образца магнезиального молока, Mg(OH)₂, определяли титрованием фосфорной кислотой (H3PO4). Для нейтрализации 30 мл магнезиального молока потребовалось 54,8 мл 0,5 М фосфорной кислоты. Уравнение этой реакции показано: 2H₃PO₄ + 3Mg(OH)₂ ⟶ 6H₂O + Mg3(PO₄)₂, Какова была концентрация магнезиального молока?

Только для подписчиков

04:22

Стенограмма видео

Концентрация образца магнезиального молока, Mg(OH)2 определяли титрованием фосфорной кислотой, h4PO4. 30 миллилитров магнезиального молока для его нейтрализации потребовалось 54,8 миллилитров 0,5-молярной фосфорной кислоты. Уравнение этой реакции показано. Два h4PO4 плюс три Mg(OH)2 реагируют с образованием шести H3O плюс Mg3(PO4)2. Какова была концентрация молоко магнезии?

Этот вопрос просит нас рассчитать концентрацию образца молока магнезии, используя результаты эксперимент по титрованию. Титрование – это количественный аналитический метод, используемый для определения концентрации аналита с использованием известного концентрация второго раствора.

Описание эксперимента по титрованию в этой задаче – кислотно-основное титрование. В ходе эксперимента фосфор кислоту, или h4PO4, добавляют в магнезиальное молоко, содержащее основание, магний гидроксид или Mg(OH)2. Реакция нейтрализации протекает место, производя воду и соль фосфата магния. Молярное отношение кислоты к основанию равно от двух до трех.

Освободим место для записи экспериментальные данные, приведенные в задаче.

Концентрация фосфорной кислоты в этом эксперименте составляет 0,5 мол. Объем фосфорной кислоты необходимое для нейтрализации молочка магнезии составляет 54,8 миллилитров. Объем молока магнезии в эксперименте 30 миллилитров. И цель этой проблемы состоит в том, чтобы рассчитать молярную концентрацию магнезии в молоке.

Решение этой проблемы требует трех шаги. Во-первых, нам нужно рассчитать число используемых молей кислоты. Затем нам нужно преобразовать моли кислоты к молям основания, используя молярное соотношение из сбалансированного уравнения. Наконец, на третьем шаге мы рассчитать молярную концентрацию основания. Прежде чем приступить к первому шагу, нам нужно перевести измерения объема из миллилитров в литры путем умножения на единицу литр на 1000 миллилитров. Давайте добавим эти значения на нашу диаграмму и начать.

Чтобы найти количество молей кислоты используется, мы можем использовать следующее уравнение, в котором число молей равно молярность в молях на литр, умноженная на объем в литрах. Умножаем 0,5 моль на литр на 0,0548 литра, что позволяет сократить единицы литры. В результате получается 0,0274 моль h4PO4.

Теперь мы можем использовать молярное соотношение из сбалансированного химического уравнения, чтобы преобразовать из молей кислоты в моли база. Умножим 0,0274 моль h4PO4 на три моля Mg(OH)2 на два моля h4PO4, что позволяет сократить из единиц молей h4PO4. В результате получается 0,0411 моль Мг(ОН)2.

Теперь, когда мы знаем количество молей прореагировавшего основания, можно рассчитать молярную концентрацию. Для расчета молярной концентрации или молярности, мы должны разделить количество молей основания на объем базы. Следовательно, нам нужно разделить 0,0411 моль на 0,03 л. В результате получается 1,37 моль/л. литр. Мы также можем представить единицы молей на литр с прописной М.

В заключении по результатам опыта по титрованию концентрация магнезиального молока 1,37 молярный.

= | Сбалансированное уравнение химической реакции

Поиск

Результаты поиска по химическому уравнению

Новости Только 5% НАСЕЛЕНИЯ знают

Рекламное объявление

1 результатов найдено
Отображение уравнения от 1 до 1 Страница 1 — Пожалуйста, прокрутите до конца, чтобы увидеть больше результатов

Уравнение Результат #1

Нажмите, чтобы увидеть более подробную информацию и рассчитать вес/моль >>

‘> Son; Фосфорная кислота; ортофосфорная кислота; Phosphoric acid hydrogen
30500 ± 0.00060″> 3Mg + 2H 3 PO 4 Mg 3 (PO 4 ) 2
магний
hydrogen magnesium phosphate
(dung dịch pha loãng) (khí) (kt)
3 2 3 1 Hệ số
Нгуен-Фантукхой (г/моль)
Со моль
Кхой лунг (г)

6 Рекламное объявление

Дополнительная информация об уравнении 3 мг + 2H

3 PO 4 → 3H 2 + MG 3 (PO 4 ) 2

Что такое условие реакции MG (Magnesium) REACTS 4PO4 (4PO4 (4PO4.

Sonac; фосфорная кислота; ортофосфорная кислота; водород фосфорной кислоты) ?

Для этого химического уравнения не найдено информации

Объяснение: идеальные условия окружающей среды для реакции, такие как температура, давление, катализаторы и растворитель. Катализаторы — это вещества, которые ускоряют темп (скорость) химической реакции, не потребляясь и не становясь частью конечного продукта. Катализаторы не влияют на равновесные ситуации.

Как могут происходить реакции с образованием h3 (водорода) и Mg3(PO4)2 (фосфата магния)?

Явление после реакции Mg (магния) с h4PO4 (Sonac; фосфорная кислота; ортофосфорная кислота; водород фосфорной кислоты)

Это уравнение не несет никакой конкретной информации о явлении.

В этом случае вам просто нужно наблюдать, чтобы убедиться, что вещество продукта Mg3(PO4)2 (фосфат магния), появляющийся в конце реакции.

Или если какое-либо из следующих реагентов h4PO4 (Sonac; фосфорная кислота; ортофосфорная кислота; водород фосфорной кислоты), исчезающий

Какую другую важную информацию вы должны знать о реакции

У нас нет дополнительной информации об этой химической реакции.

Формулы перестановки сочетания размещения: Перестановки, размещения и сочетания. Формулы.

Формулы комбинаторики — Дискретная математика (Математика)

2.1. Формулы комбинаторики

2.1.1. Перестановки    

1) Перестановки без повторений.

Перестановки — это комбинации, состоящие из одних и тех же элементов и отличающиеся только порядком расположения этих элементов. Возьмем n различных элементов a1, a2, a3, … an; будем переставлять эти элементы всевозможными способами, оставляя без изменения число элементов и меняя только порядок их расположения. Обозначим общее число полученных таким образом перестановок P(n). P — первая буква французского слова permutation — перестановка.

Составив таблицу перестановок для n элементов и применив (n — 1) раз правило произведения, получим число всех возможных перестановок:

P(n) = n • (n -1) • (n — 2) • … • 3 • 2 • 1 = n!

Такие перестановки называются перестановками без повторений (один и тот же элемент не может повториться в комбинации, все элементы различны).

Задача: шесть человек могут в разном порядке сесть за круглый стол, сколько существует способов разместить эти шесть человек за столом?

Решение: т.к. все люди различны и  их комбинации различаются только порядком следования, то мы имеем перестановки без повторений. Определим их число:

Р(6) = 6! = 1 • 2 • 3 • 4 • 5 • 6 = 720.

2) Перестановки с повторениями

Рассматривая различные перестановки, мы предполагали, что все n элементов различны. Если же некоторые элементы повторяются, то в этом случае комбинации с повторениями вычисляют по другим формулам.

Если среди n элементов есть n1 элементов одного вида, n2 элементов другого вида и т.д., nk элементов к-го вида, то имеем перестановки с повторениями, их число:

, где n1+…+nk = n.

Задача: сколько различных «слов» можно составить из букв слова ДЕД, МАТЕМАТИКА.

Решение: имеем перестановки с повторениями.

А) ДЕД    n=3, k=2, n1=2, n2=1

P3(2, 1) = 3!/(2! • 1!) = 6 / 2 = 3;

Б) МАТЕМАТИКА n=10, k=6, n1=2, n2=3, n3=2, n4=n5=n6=1

P10(2,3,2,1,1,1)=10!/(2! • 3! • 2!)=2•4•5•6•7•9•10 = 134 400.


2.1.2. Размещения

1) Размещения без повторений.

Размещениями называют комбинации, составленные из n данных элементов по k элементов (k<=n, k>0), которые отличаются либо составом элементов, либо порядком расположения элементов. Обозначаются размещения Ank . А — первая буква французского слова arrangement, что в переводе означает «размещение», «приведение в порядок». Число всех возможных размещений находится по формуле:

.

Задача: расписание одного дня состоит из двух пар. Определить число вариантов расписания при выборе из пяти дисциплин, если не может быть одинаковых пар.

Решение: имеем размещения без повторений из пяти элементов по два, из число: .

2) Размещения с повторениями.

Пусть существуют n различных элементов. Выберем из них m штук, действуя по следующему принципу: возьмем любой элемент, но не будем устанавливать его в какой-либо ряд, а просто запишем под номером 1 его название, сам же элемент вернем к остальным элементам. Затем опять из всех n элементов выберем один, запишем его название под номером 2 и снова вернем элемент обратно. Будем выполнять эти операции, пока не получим m названий. Размещения с повторениями вычисляются по формуле:

.

Задача: сколько четырехзначных номеров можно составить из 10 цифр?

Решение: имеем размещения с повторениями из 10 элементов по 4, их число: .


2.1.3. Сочетания

1) Сочетания без повторений.

Сочетаниями называют комбинации, составленные из n различных элементов по k (k =< n) элементов, которые отличаются хотя бы одним элементом. Сочетания обозначаются: Cnk C — первая буква французского слова combinasion — сочетание.

Составим из n элементов всевозможные сочетания по k элементов в каждом. Их будет Cnk . Внутри каждого сочетания, состоящего из k элементов, образуем всевозможные комбинации, учитывающие порядок следования в них элементов. Таких комбинаций будет Pk, т. к. мы в нашем сочетании образовываем перестановки. Всего различных комбинаций из n элементов по k в каждой, отличающихся друг от друга либо составом (элементами), либо порядком их следования, будет Cnk • Pk . Но такие комбинации называются размещениями. Таким образом, Ank = Cnk • Pk, тогда:

.

Рекомендация для Вас — 4.2. Методы статистического анализа.

Задача: в шахматном турнире участвует 7 человек; сколько партий будет сыграно, если между любыми двумя участниками должна быть сыграна партия?

Решение: имеем сочетания без повторений из 7 элементов по 2; их число:  .

2) Сочетания с повторениями.

Если в сочетаниях некоторые элементы (или все) могут оказаться одинаковыми, то такие сочетания называются сочетаниями с повторениями. Их число определяется по формуле:                               .

Задача: сколько наборов из 7 пирожных можно составить, если в продаже имеется 4 сорта пирожных?

Решение: имеем сочетания с повторениями из четырех по 7 по, их число: .

Элементы комбинаторики, размещения, перестановки, сочетания. Упорядоченное множество. Случайные события, алгебра событий, формулы де Моргана

Математика \ Математика

Страницы работы

55 страниц (Word-файл)

Посмотреть все страницы

Скачать файл

Фрагмент текста работы

1.  Элементы комбинаторики, размещения, перестановки, сочетания.

Упорядоченное множество – это множество с заданным порядком элементов.

Правило произведения: Пусто А некоторое множество. Пусть элемент а1А можно выбрать n1 способами. После этого а2 можно выбрать n2 способами и тд. Тогда одновременный выбор а1, а2…аn можно сделать n1*n2…*nk способами/

Правило суммы: Пусто А некоторое множество. Пусть элемент а1А можно выбрать n1 способами. После этого а2 можно выбрать n2 способами и тд. Тогда выбор одного из элементов (а1 или а2 или аn) можно сделать n1+n2+…+nk способами.

Размещение из n по k: Размещением из n по k называется каждое упорядоченное подмножество из к элементов, выбранных из n элементов

Перестановка – упорядоченное множество

Сочетания – подмножестов из к элементов, выбранных во множестве из n переменных

1)            2)


2. Случайные события, алгебра событий, формулы де Моргана.

Событие – исход эксперимента

Эксперимент – создание определенного комплекса условий

Пространство событий – совокупность событий при данном эксперименте

Ø – пустое подмножество(невозможное событие)

Ω — все множество (достоверное событие)

Несовместные события – события А и В называются несовместными, если они не могут наступить вместе

Противоположные события – это совокупность событий, противоположных А. (дополнение)

Алгебра событий

А+В=АВ  wA+B ówA V wB

A·B=AB  wA·BówAwB

Свойства операций:

1)  А+В=В+А  2) А*В=В*А  3) А+ =Ω  4) А*Ω=А  5)АВ С А  6) *А= Ø  7) =А   8) А-В=А*  9)   10)


3.Классическая вероятность, теорема сложения вероятностей.

Вероятность события – это число, характеризующее возможность наступления данного события P(A)

Вероятность невозможнгого=0, а достоверного=1. Вероятность любого другого [0,1].

Классическая вероятность – это классическое пространство событий, состоящее из конечного числа элементарных событий и все они равновозможны. P(A)=na/n, где na-благоприятные исходы, n- все возможные исходы

Теорема сложения вероятностей

Если А и В два события, то вероятность их суммы P(A+B)=P(A)+P(B)-P(AB)

Для несовместных событий:

P(A+B)=P(A)+P(B)

Для 3-х: P(A)+P(B)+P(C)-P(AC)-P(AB)-P(BC)+P(ABC)

P()=1-P(A)


4.   Статистическое определение вероятности, геометрическая вероятность.

Пусть при проведении n испытаний, некоторое событие А появилось m раз.Многочисленные эксперименты такого рода показывают, что при больших n отношение m/n, называемое частостью события А, остается примерно постоянным. Статистическое определение вероятности заключается в том, что за вероятность события А принимается постоянная величина, вокруг которой колеблются значения частостей при неограниченном возрастании числа n.

Геометрическая вероятность

Обобщением классической схемы является пространство событий, элементарные исходы которого можно представить в виде точек, заполняющих некоторою область Ω в (трехмерном) пространстве в R3 . Если при этом  событию А благоприятствуют элементарные события, заполняющие некоторую подобласть D из Ω, то геометрической вероятностью события А  называется отношение объема области D к объему области Ω: P(A)=V(D)/V(Ω). Аналогично определяется геометрическая вероятность события, когда множество Ω представляет собой некоторую область на плоскости или отрезок на прямой линии. В этих случаях объемы областей заменяются , соответственно, площадями фигур или длинами отрезков.


5. Аксиоматическое определение вероятностей.

Ω={ω} —  набор элементарных событий

Рассмотрим набор подмножеств F={A/A c Ω}

Аксиома 1: Если А1, А2…Аn…  F => A1+A2+…+An+…  F

Аксиома 2:  Для любого АF =>   F

Любой набор подмножеств данного множества, обладающий свойствами 1 и 2 – σ-алгебра

Из этих аксиом следует, что произведение любого подмножества тоже принадлежит σ-алгебре.

Аксиомы вероятностей

Вероятность события АF(A c Ω) называется число  P(A), которое обладает следующими свойствами:

1)  P(Ω)=1 P(A)>=0

2)  Если семейство событий А1, А2…Аn… попарно несовместны, т. е. Ai*Aj=Ø, то вероятность суммы  P(A1+A2+…+An+…)=P(A1)+P(A2)+…+P(An)+…

Дискретное вероятностное пространство —  вероятностное пространство, в  котором пространство событий конечно или счетно, а σ-алгебра F  — множество всевозможных подмножеств дискретного множества Ω .

6.Условные вероятности и теорема умножения.

На практике часто встречаются ситуации, когда наступление некоторого события значительно меняет возможности наступления других событий и их вероятности. Если произошло событие B, то новая вероятность события А называется условной вероятностью и обозначается PB(A), говорят: «вероятность события А при условии B». При этом B оказывается достоверным событием и играет роль пространства элементарных событий Ω.

Условная вероятность PB(A) определяется формулой (при P(B) >0):

PB(A)= (1)

Из (1) следует формула:

P(A*B)= PB(A) P(B), P(B)>0,  (2)

И симметричная формула:

P(A*B)= PA(B)P(A), P(A)>0     (3)

Независимыми называются такие события

Похожие материалы

Информация о работе

Скачать файл

Комбинации и перестановки – подготовка к экзамену Каплана

Давайте попрактикуемся в математике GRE. Проблемы с комбинациями и перестановками часто заставляют учащихся задуматься, с чего же начать. Знание уравнения для каждой операции полезно, но недостаточно — вы также должны быть в состоянии определить, какая формула необходима для ответа на поставленный вопрос.

Комбинации и перестановки на GRE

Эмпирическое правило состоит в том, что комбинации неупорядочены, а перестановки упорядочены, но что это значит? Нам нравится иллюстрировать разницу с помощью социального клуба.

  • Представьте, что в социальном клубе 10 разных членов, и вас спросят: » Сколько групп из 3 членов вы можете выбрать из социального клуба, чтобы создать партийный комитет? ” Вам нужно было бы делать комбинации или перестановки, чтобы сформулировать ответ? Откуда вы знаете?
  • В качестве альтернативы, представьте, что мы немного изменим вопрос и спросим: «  Офицерский состав состоит из президента, вице-президента и казначея. Сколько разных офицерских списков вы можете выбрать из членства в социальном клубе? ” Это тот же вопрос? Или это другое? Вам нужно использовать комбинации или перестановки?

Вопросы, на самом деле, совсем другие. Итак, как вы применяете каждый метод на GRE?

Решение задач с комбинациями

Первый вопрос (« Сколько групп из 3… ») указывает на то, что мы считаем группы из 3 человек, и нам не нужно беспокоиться о том, кого выбрать первым, вторым или третьим, т. е. порядок не имеет значения. По этой причине это проблема комбинаций.
Чтобы ответить на вопрос, мы будем использовать формулу комбинаций, где n  = общее количество элементов (10) и k  = количество выбранных элементов (3). Обратите внимание, что k может равняться n , но никогда не может быть больше n (мы можем выбрать все элементы в группе, но не можем выбрать больше элементов, чем общее количество). Вот формула комбинаций:

Обратите внимание, что восклицательный знак означает факториал; Факториал означает умножение числа на каждое целое число под ним до 1. Например, 4! = 4 * 3 * 2 * 1.
Подставив наши значения в уравнение, мы получим следующее (убедитесь, что вы уменьшили числа в расширенных вычислениях, чтобы упростить фактическое умножение, которое вам нужно сделать):

Таким образом, мы могли выбрать 120 различных групп из 3 партийных комитетов.

Решение задач перестановки

Второй вопрос спрашивает: « Сколькими различными способами вы можете выбрать список офицеров из 3 человек? ”Эта формулировка говорит нам, что мы должны отслеживать каждый выбор независимо, а не группами по 3 человека. Например, выбор Ника в качестве президента, затем Ким в качестве вице-президента, затем Приянки в качестве казначея – это не то же самое, что выбор Ким в качестве президента, затем Приянка в качестве вице-президента, затем Ник в качестве казначея, что не то же самое, что выбрать Ким в качестве президента, затем Ника в качестве вице-президента, затем Приянку в качестве казначея и так далее, то есть порядок имеет значение. По этой причине это проблема перестановок. Чтобы ответить на этот вопрос, воспользуемся следующей формулой перестановок:

Как видите, знаменатель — это точка различия между формулами комбинаций и перестановок. Для любых значений n и k количество комбинаций, которые мы можем составить, всегда будет меньше, чем количество перестановок, которые мы можем составить. Эта проблема не исключение. Подставив наши значения в уравнение, а затем максимально уменьшив, мы получим:

Таким образом, когда порядок имеет значение и мы отслеживаем каждый выбор по-разному, мы можем выбрать 3 офицеров 720 различными способами.

Разработчики тестов GRE создают сложные задачи, используя тонкий язык, чтобы указать, следует ли вам использовать формулу комбинации или перестановки для ответа на поставленный вопрос. Комбинированные вопросы укажут на то, что вам нужно сформировать группы или наборы; Вопросы перестановки будут содержать слова или фразы, указывающие порядок, например «первый, второй, третий» или «сколько разных способов». Некоторые действительно сложные задачи могут сочетать в себе и то, и другое.
Как гласит старая поговорка, «практика делает совершенным» — чем больше этих задач вы решите (и чем больше соответствующих объяснений вы прочитаете), тем лучше вы будете подготовлены к ответам на вопросы о комбинациях и перестановках в день экзамена GRE.

Перестановка/комбинация — GRE Math

Все математические ресурсы GRE

13 Диагностические тесты 452 практических теста Вопрос дня Карточки Learn by Concept

← Предыдущая 1 2 3 4 5 Следующая →

GRE Math Help » Арифметика » Перестановка/комбинация

Правление торговой палаты состоит из семи членов, выбранных из двадцати кандидатов. Процесс выборов в совет состоит из двух этапов. Сначала избираются президент, секретарь и казначей. После этого четыре члена выбираются «на свободе» без какого-либо конкретного титула или округа. Сколько возможных досок можно было выбрать?

Possible Answers:

390,700,800

16,279,200

5,426,400

10,465,200

2,713,200

Correct answer:

16,279,200

Объяснение:

Мы должны рассмотреть два случая. Во-первых, группа, состоящая из президента, секретаря и казначея, представляет собой случай перестановки. Поскольку в такой группе порядок имеет значение, мы можем выбрать из наших первоначальных 20 кандидатов 20*19* 18 или 6840 возможных группировок.

После этого группа в целом представляет собой случай комбинаций, в которых порядок не имеет значения. Поскольку мы уже выбрали 3 на первые три слота, останется 17 человек. Формула выбора набора из 4 человек из 17 кандидатов представлена ​​формулой комбинации такого вида:

17! / ((17-4)! * 4!) = 17! / (13! * 4!) = (17 * 16 * 15 * 14) / (4 * 3 * 2) = 17 * 4 * 5 * 7 = 2380

Таким образом, мы имеем 6840 и 2380 возможных группировок. Каждый из них можно комбинировать друг с другом, что означает, что у нас есть 6840 * 2380, или 16 279.200 потенциальных досок.

Сообщить об ошибке

Сколько заказанных образцов из 5 карт можно взять из колоды из 52 карт без замены?

Possible Answers:

42,365,000

311,875,200

500,320,000

11,452,000

65,690,040

Correct answer:

311,875,200

Объяснение:

Ключевые моменты, которые мы должны помнить, это то, что порядок имеет значение и что мы делаем выборку без замены. Тогда это становится простой проблемой перестановки. У нас есть 52 карты, которые нужно выбрать по 5 за раз, поэтому ответ 52 * 51 * 50 * 49.* 48.

Сообщить об ошибке

Количественное сравнение

Из стандартной колоды в 52 карты выбираются 3 карты.

Количество A: Количество способов выбрать 3 карты с заменой

Количество B: Количество способов выбрать 3 карты без замены

Возможные ответы:

Связь не может быть определена из предоставленной информации.

Количество B больше.

Количество A больше.

Обе величины равны.

Правильный ответ:

Количество A больше.

Объяснение:

Количество A говорит  с заменой , поэтому у нас есть 52 способа выбрать первую карту, затем мы заменяем ее, так что у нас снова есть 52 способа выбрать вторую карту, и аналогично у нас есть 52 способа выбрать 3-ю карту. . Таким образом, у нас есть 52 * 52 * 52 способа выбора 3-х карт с заменой.

Количество B говорит без замены , поэтому у нас есть 52 способа выбрать первую карту, но затем мы не кладем эту карту обратно в колоду, поэтому у нас есть 51 способ выбрать вторую карту. Мы снова не кладем эту карту обратно, оставляя 50 способов выбрать третью карту. Таким образом, есть 52 * 51 * 50 способов выбрать 3 карты без замены.

Очевидно, количество A больше. В общем, всегда должно быть больше способов выбрать что-то с заменой , чем без замены , как мы показали выше. Если бы вы уже знали это, вы могли бы выбрать количество А без математики. Обратите внимание, что в любом случае вы придете к ответу здесь, нет НИКАКОЙ причины заканчивать вычисления полностью. Это сэкономит вам время при решении многих задач количественного сравнения. Например, мы знаем, что 52 * 52 * 52 больше, чем 52 * 51 * 50, но на самом деле не выяснили, чему равны эти два выражения.

Сообщить об ошибке

Сколькими способами можно посадить 6 розовых кустов разного цвета в ряд, в котором есть место только для 4 кустов?

Возможные ответы:

120

480

240

220

360

Правильный ответ:

360

Пояснение:

Есть 6 способов выбрать первый розовый куст, 5 способов выбрать второй, 4 способа выбрать третий и 3 способа выбрать четвертый. Всего есть 6 * 5 * 4 * 3 = 360 способов расположить кусты роз.

Сообщить об ошибке

Сколькими способами можно расположить в ряд пять разноцветных шаров?

Возможные ответы:

Правильный ответ:

Пояснение:

Нам нужно расставить 5 шаров по 5 позициям: _ _ _ _ _. Первую позицию может занять любой из 5 шаров. Затем остается 4 шара, чтобы заполнить вторую позицию, и так далее. Поэтому количество аранжировок равно 5! = 5 * 4 * 3 * 2 * 1 = 120.

Сообщить об ошибке

Дейзи хочет поставить четыре вазы в ряд за пределами своего сада. У нее есть восемь ваз на выбор. Сколько ваз она может сделать?

Возможные ответы:

Правильный ответ:

Объяснение:

В этой задаче, поскольку порядок ваз имеет значение (красный синий желтый отличается от синего красного желтого), мы имеем дело с перестановками.

При выборе из возможных вариантов общее количество возможных перестановок (порядок имеет значение):

Сообщить об ошибке

Какое минимальное количество рукопожатий может произойти между пятнадцатью людьми на собрании, если каждый человек пожимает друг другу руку только один раз?

Возможные ответы:

105

250

210

32 760

Правильный ответ:

105

Объяснение:

Это комбинированная задача вида «15 выбирают 2», потому что наборы рукопожатий не имеют значения по порядку. (То есть «A пожимает руку B» — это то же самое, что «B пожимает руку A».) Используя стандартную формулу, получаем: 15!/((15 – 2)! * 2!) = 15!/(13! * 2!) = (15 * 14)/2 = 15 * 7 = 105,

Сообщить об ошибке

20 человек имеют право быть избранными в городской совет, в который входят трое избираемых членов.

Количество A

Количество возможных комбинаций членов совета при условии отсутствия различий между должностными лицами.

Количество B

Количество возможных комбинаций членов совета, учитывая, что в совете есть президент, вице-президент и казначей.

Возможные ответы:

Количество B больше.

Количества равны.

Количество A больше.

Связь не может быть определена на основании предоставленной информации.

Правильный ответ:

Количество B больше.

Объяснение:

Это вопрос перестановок и комбинаций. Вы можете решить это, используя соответствующие формулы, но всегда случай, когда вы можете сделать больше перестановок, чем комбинаций для всех групп размером больше единицы, потому что порядок выбора имеет значение; поэтому, не занимаясь математикой, вы знаете, что ответом должно быть B.

Сообщить об ошибке

У Джо есть набор из 10 книг, которые он еще не читал. Если он возьмет 3 из них в отпуск, сколько возможных наборов книг он сможет взять?

Возможные ответы:

1145

240

Ни один из этих

720

120

Правильный ответ:

120

Объяснение:

Он может выбрать из 10, затем 9, затем 8 книг, но поскольку порядок не имеет значения, нам нужно разделить на 3 факториал

 (10 * 9 * 8) ÷ (3 * 2 * 1) = 720/6 = 120

Сообщить об ошибке

Сколько различных лицензионных паролей можно составить, если указанный пароль должен содержать ровно 6 символов, два из которых являются различными числами, одно из которых должно быть заглавной буквой, а остальные 3 могут быть любой цифрой или буквой (заглавной или строчной), чтобы в пароле не было повторений каких-либо символов?

Возможные ответы:

619652800

219

231

456426360

365580800

Правильный ответ:

456426360

Объяснение:

Начните с рассмотрения трех «жестких и быстрых условий» — цифр и одной буквы верхнего регистра.

4 в минус 7 степени: Калькулятор степеней — возвести в степень онлайн

Ультимативный дайджест по ИИ и нейросетям для дизайнера и не только — Дизайн на vc.ru

3201 просмотров

Введение

Всем привет! Меня зовут Артем Говердовский, я продуктовый дизайнер и, так уж получилось, что я люблю все структурировать, собирать и раскладывать любые материалы «по полочкам», и это касается (к сожалению или к счастью) не только дизайна.

К этому дайджесту я подходил достаточно долго, так как инструментарий ИИ (Искусственный интеллект, с англ. ai — Artificial intelligence) применительно к сфере дизайна, развивается достаточно быстро (об этом я, кстати, делился мыслями с РБК Тренды в статье «Зачем нужны технологии искусственного интеллекта и нейросети в дизайне»)

С учетом того, что ждать пока выйдут ВСЕ инструменты завязанные на ИИ — бессмысленное занятие, я решил поделиться уже теми, что у меня накопились, и теми, что я успел попользоваться. Инструментарий будет полезен дизайнерам (продуктовым ui/ux, графическим, моушн), дизайн-менеджерам, видео/аудио монтажёрам, SMM-менеджерам и всем, кто хочет сэкономить время воспользовавшись инструментами ИИ.

Оглавление

Для удобства, разобьем весь инструментарий ИИ на разделы:

  1. Инструменты ИИ преобразующие текстовые запросы в иллюстрацию/рисунок/фото (text-to-image)
  2. Инструменты ИИ преобразующие текстовые запросы в видео (text-to-video)
  3. Инструменты ИИ для работы видео
  4. Инструменты ИИ для работы с фото/изображениями/3D
  5. Инструменты ИИ для работы с аудио
  6. Инструменты ИИ для работы с текстом

Часть сервисов, я «засунул» сразу в несколько разделов, так как они подходят под тематике и туда и туда.

Инструменты ИИ преобразующие текстовые запросы в иллюстрацию/рисунок/фото (text-to-image)

Такие инструменты ИИ, в которых можно написать текстовой запрос, как в поисковой строке гугла/яндекса, а на выходе получить изображение/фото/иллюстрацию

Нейросеть, которая генерирует изображения по текстовому описанию. Важное отличие Stable Diffusion от Dalle2 и Midjourney в том, что она является нейронкой с открытым исходным кодом и её можно поставить себе на комп и оффлайн сколько хочешь + тут можно сразу в Dezgo попользоваться данным ИИ + тут также можно попробовать + тут тоже можно попробовать.

Одна из самых популярных нейросетей, генерирующих изображения по текстовому описанию. Я и сам, преимущественно, пользуюсь сейчас ей, так как она удовлетворяет текущие потребности.

Парочка статей, из последнего, что позволит понять как она работает:

• Огромная библиотека стилей для Midjourney

• Обзор обновлённой Midjourney V4

• Инструмент помогающий собрать промпт фразу

Куда же без него. Данный сервис точно у многих на слуху когда речь заходит про text-to-image. И его мини брат DALLEmini.

Аналог DALL·E 2 от отечественных разработчиков с возможностью писать запрос на русском.

Еще один ИИ сервис text-to-image, в котором можно выбрать в каком стиле вы хотите увидеть исходную иллюстрацию.

Также неплохой инструмент, внутри есть опции и параметры, при которых нейронка сможет обработать запрос в разных стилях, или взять за основу референс, предоставленный пользователем.

Нейросеть от Meta, которая оживляет детские рисунки. Просто загружаем рисунок (рисунок должен быть на белом листе бумаги и хорошо освещён) и выбираем настройки анимации

Обучаемый ИИ на примерах. Можно натренировать ИИ на каком-то специфичном объекте (например ваше фото), чтобы потом получать результат, основанный на данном объекте.

Нейросеть, которая поможет повысить качество фото, аудио и видео + генерирует картинки по текстовому запросу.

Инструменты ИИ преобразующие текстовые запросы в видео (text-to-video)

Все тоже самое что и и выше, только тут уже текстовой запрос преобразуется ИИ не в фото/иллюстрацию, а в видео.

На самом деле, помимо данной функции (text-to-video) у инструмента еще много других:

• Быстро отфильтрует в видео нужно и создаст короткий ролик/трейлер/рилс

• Найдет ключевые мысли и точки в видео и сделает «выжимку» из видео-контента

• Напишет сценарий к видео, разобьет его на сцены и прочее

• и много еще чего. ..

Поможет создать аватар по текстовому запросу, озвучить его и анимировать.

По факту — ИИ студия. Огромный потенциал и экономия для тех, кто создает видео контент. Единственный минус — полным функционалом, можно пользоваться только после оплаты.

Инструменты ИИ для работы с видео

Инструменты, которые помогают при работе с видео. Не путать с text-to-video, где мы через текстовой запрос, можем создать видео с нуля. В данном случае, мы говорим про инструменты ИИ, которые помогают с обработкой/редактированием/цветокором видео файлов.

Цветокоррекция видео за счет автоматизации подбора и балансировки цветов на базе искусственного интеллекта.

Апскейлит видео в разные разрешении (особенно 4К) с применением нейросетей улучшая битрейт и качество картинки.

Сервис на основе ИИ, который раскрашивает черно-белые фото и видео.

Нейросеть, которая поможет повысить качество фото, аудио и видео + генерирует картинки по текстовому запросу.

Видеоредактор на базе ИИ, позволяет убрать фон/объекты из видео, а также заменить их на другие + масса других возможностей.

Инструменты ИИ для работы с фото/изображениями/3D

Все, что помогает с обработкой/редактированием/цветокором/улучшением качества при работе с фото/изображениями/3D

Апскейлит фото/изображение и улучшает его качество.

Сервис для «оживления», улучшения и цветокора (из ч/б в цвет) фото.

Нейросеть, которая за пару секунд позволяет удалить фон с фотографии.

Сервис, который позволяет быстро:

  • Убрать фон;
  • Вырезать объект;
  • Улучшить качество;
  • Добавить источник света.

Бесплатный бот в телеграм для улучшения фото

Нейросеть, которая поможет повысить качество фото, аудио и видео + генерирует картинки по текстовому запросу.

Cервис, который превращает изображения в простые анимации (можно выгрузить в MP4 и GIF).

Инструменты ИИ для работы с аудио

Все, что помогает с обработкой/редактированием/улучшением качества аудио файлов.

Генерация музыки из текста с помощью ИИ.

Работа с голосом, озвучкой и др. Зачитывает любой текст, имитируя голоса знаменитостей — от Эминема и Канье Уэста до Тупака. Помимо музыкантов на сайте доступны политики и телеведущих.

Именно с помощью этой нейросетки было создано аудио, в котором блогер озвучил текст голосом репера Дрейка, сгенерировав тем самым с нуля песню про бобы)

Cервис, который сделает минусовку из любой песни. Нейросеть разделяет песню на две отдельные дорожки: чистый вокал и инструментальная партия.

Генератор голоса на основе ИИ. Прочитает написанный вами текст голосами актёров озвучки. Идеально подойдёт для озвучивания видеоконтента.

Нейросеть, которая поможет повысить качество фото, аудио и видео + генерирует картинки по текстовому запросу.

Очень необычная нейросеть, как и все японское. Позволяет получить звуковое сопровождение к любой картинке. Вы загружаете фотографию, а алгоритм анализирует содержимое и подбирает саундтрек. Можно не только выбирать свои селфи, но и ткнуть на случайную локацию на картах.

Бесплатный сервис от Adobe, который значительно улучшает качество аудиозаписей. Инструмент способен довести запись, сделанную на средний микрофон, до такой степени, что она будет звучать как выполненная в профессиональной студии.

Инструменты ИИ для работы с текстом

Все, что помогает с редактированием/созданием/преобразованием текстового контента.

Российский сервис по созданию заголовков, уникальных описаний, постов, комментариев и прочее.

Последнее время очень на слуху. И не зря — можно беседовать и получать развернутые ответы на почти любые вопросы. Писать код, проводить редактуру и корректуру текста, писать стихи, попросить сократить статью до списка ключевых мыслей. Можно свой список ключевых мыслей развернуть в статью. Попросить решить домашку или написать реферат. В общем, возможностей масса. + Гайд для РФ, регистрации ChatGPT

ИИ от Яндекса, продолжает вашу фразу, генерируя текст.

Текстовый ИИ, который пишет для вас маркетинговые текста и описания по вашим параметрам и ключевым словам.

Почти как ChatGPT, но от Notion. Пока запущенна в тестовом режиме, инвайта надо ждать оооочень долго. Но для тех, кто работает в Notion — вещь полезная.

На этом у меня все. Спасибо, что долистали до конца! Если есть желание получать больше контента о Искусственном Интеллекте (A.I.), нейросетях, UI/UX, VR|AR, технологиях, софте, метриках, исследованиях, психологии дизайна, дизайн-менеджменте, то велком в мой телеграм-канал ДЗГН’360

Ну и, конечно, жду комментах ваших любимых ИИ инструментов, которые помогают вам в работе/хобби 😎

P.s. Для тех, кто хочет еще больше инструментов на основе ИИ:

  • FUTUREPEDIA — Большая обновляемая библиотека всевозможных инструментов на основе ИИ.
  • huggingface — Репозиторий ИИ проектов, много можно найти демок и бесплатных версий.
3-8 9 Оценить квадратный корень из 12 10 Оценить квадратный корень из 20 11 Оценить квадратный корень из 50 94 18 Оценить квадратный корень из 45 19 Оценить квадратный корень из 32 20 Оценить квадратный корень из 18 92

Математические выражения: Примеры экспоненциальных законов

00:00:01. 090
В этом уроке представлены примеры вопросов по экспоненциальным законам. Теперь упростим следующее.

00:00:09.200
Рассмотрим этот пример, 4 в степени 3 умножить на 2 в степени 4.

00:00:16.190
Теперь, если мы рассмотрим этот закон, мы поймем не может использовать его, потому что базы для этих терминов не совпадают.

00:00:24.190
Следовательно, нам нужно найти способ сделать эти базы одинаковыми.

00:00:29,170
Для этого заметьте, что 4 здесь равно, 2 умножить на 2, что также равно, 2 в степени 2.

00:00:39,130 ​​
Следовательно, мы можем замените 4 на 2 в степени 2.

00:00:44.130
Хорошо, обратите внимание, что для продолжения нам нужно упростить этот термин.

00:00:49.150
Для этого мы можем использовать этот закон.

00:00:53.180
Прежде чем использовать его, давайте сначала подберем цвета.

00:00:58.060
Теперь 2 в степени 2, в степени 3 равно, 2 в степени 2 умножить на 3.

00:01:06.150
2 умножить на 3 дает 6.

00:01:12.040
Теперь мы можем использовать этот экспоненциальный закон для дальнейшего упрощения этих терминов. Давайте сначала подберем цвета.

00:01:20.110
2 в степени 6 умножить 2 в степени 4, получится 2 в степени 6 плюс 4.

00:01:27.050
Теперь 6 плюс 4 дает 10. Это самый простой термин. Итак, ответ равен 2 в степени 10.

00:01:38,200
Следующий пример, давайте упростим.

00:01:42.140
Для этого мы можем использовать этот экспоненциальный закон.

00:01:46:040
Теперь 3 в степени 2 в степени 3 равно, 3 в степени 2 умножить на 3.

00:01:53.240
2 умножить на 3 дает 6.

00:01:58.240
Далее упростим этот термин.

00:02:02.080
3 в степени минус 3, в степени 4 равно, 3 в степени минус 3 умножить на 4.

00:02:10.150
Теперь, умножение минус 3 на 4 дает минус 12.

00:02:17.000
Для дальнейшего упрощения мы можем использовать этот экспоненциальный закон.

00:02:21:110
Давайте сначала подберем цвета.

00:02:25.010
Теперь, используя экспоненциальный закон для упрощения этих терминов, мы получаем 3 в степени 6 плюс отрицательное 12.

00:02:33.070
Обратите внимание, что умножение положительного на отрицательное дает отрицательное значение.

00:02:39.130
Таким образом, когда мы умножаем эти члены, мы получаем 3 в степени 6 минус 12.

00:02:46.060
6 минус 12 дает отрицательное число 6. Теперь у нас есть 3 в степени отрицательного 6.

00:02:52.220
Лучше не оставлять ответ в виде отрицательного показателя степени. Итак, давайте изменим это на положительный показатель степени.

00:03:00.160
Теперь обратимся к закону экспоненты. Мы видим, что 3 в степени минус 6 равно, 1 делится на 3 в степени 6.

00:03:13.240
Хорошо, следующий пример.

00:03:16.100
Упростим этот термин.

00:03:19.200
8 делит на 4 дает 2. Теперь, обращаясь к этому экспоненциальному закону, мы можем видеть, что p в степени 5 делится на p в степени 2 дает p в степени 5 минус 2.

Как найти площадь треугольника через медианы: Нахождение площади через медианы

Медиана равностороннего треугольника – формула

4.6

Средняя оценка: 4.6

Всего получено оценок: 126.

4.6

Средняя оценка: 4.6

Всего получено оценок: 126.

Равносторонний треугольник стоит особняком среди всех фигур: в нем легко можно найти значение всех сторон и углов, так как все углы известны заранее, а найдя одну сторону, можно найти сразу все три. Но именно из-за этих свойств, составители задач любят писать каверзные условия, в которых не всегда можно разобраться с первого раза, например, не всегда можно понять, что такое медиана, потому что человеку проще воспринимать понятие высоты, нежели медианы. Рассмотрим же понятие медианы в равностороннем треугольнике подробно.

Определения

Равносторонний треугольник – это треугольник, все стороны которого равны, а углы по 60 градусов.

Равносторонний треугольник это частный случай равнобедренного, но в равностороннем любую сторону можно считать основанием.

Рис. 1. Равносторонний треугольник.

Из этого следует, что любая высота равностороннего треугольника является медианой и биссектрисой, так как любая высота проводится к стороне, которую можно считать основанием.

Медиана – это отрезок, соединяющий вершину треугольника с серединой противоположно стороны. Медиана также имеет ряд свойств, которые можно использовать в решении задач.

Медианы в треугольнике пересекаются в одной точке и делят эту точку в отношении 2:3, считая от вершины. При этом медианы разбивают треугольник на 6 разновеликих треугольников. Если посмотреть на рисунок, то можно увидеть, что в равностороннем треугольнике каждый из 6 этих треугольников будет прямоугольным.

Формула медианы равностороннего треугольника

Выведем формулу медианы равностороннего треугольника. В равностороннем треугольнике АВС проведем высоту АН. Она же будет являться медианой и высотой. Медиана разобьет треугольник на два прямоугольных: АНС и АНВ. Рассмотрим треугольник АНС.

Рис. 2\over4}}$$

Это и есть формула медианы равностороннего треугольника. С другой стороны, можно воспользоваться тригонометрическими тождествами и вывести еще одну формулу:

$$sin(ACH)={AH\over AC}$$

При этом угол АСН равен 60 градусам. Значит, можно определить синус угла: $$sin(ACH)={\sqrt{3}\over 2}$$

Выразим значение медианы АН

$$АН=sin(ACH)*AC={\sqrt{3}\over2}*AC={\sqrt{3}\over2}*a$$

Вот еще одна формула, характерная для равностороннего треугольника.

Задача

Для закрепления темы решим простую задачу на обратное использование формулы медианы.

В равностороннем треугольнике медиана равна $$20\over{\sqrt{3}}$$. Найти площадь треугольника.

Для нахождения площади воспользуемся классической формулой.

Классическую формулу можно использовать для нахождения площади любого треугольника.

Для нее нам нужно значение стороны и высоты. Высота в равностороннем треугольнике совпадает с медианой, поэтому нужно найти только сторону. Выразим ее через формулу медианы равностороннего треугольника.

Рис. 3. Рисунок к задаче.

$$m={\sqrt{3}\over2}*a$$

$$a={m\over{\sqrt{3}\over2}}=m*{2\over\sqrt{3}}$$

Подставим в формулу значение медианы:

$$a={20\over\sqrt{3}}*{2\over\sqrt{3}}={40\over3}$$

Посчитаем площадь:

$$S={1\over2}*a*m={1\over2}*{40\over3}*{20 \over\sqrt{3}}={400\over{3\sqrt{3}}}$$

Что мы узнали?

Мы вывели две формулы медианы равностороннего треугольника, дали определения, необходимые для решения задач и решили небольшую задачу для закрепления знаний.

Тест по теме

Доска почёта

Чтобы попасть сюда — пройдите тест.

  • Александр Рудаков

    5/5

Оценка статьи

4.6

Средняя оценка: 4.6

Всего получено оценок: 126.


А какая ваша оценка?

Как найти площадь треугольника: прямоугольного, равнобедренного и тд

Sign in

Password recovery

Восстановите свой пароль

Ваш адрес электронной почты

MicroExcel. ru Математика Геометрия Нахождение площади треугольника: формула и примеры

Треугольник – это геометрическая фигура, которая состоит из трех сторон, образованных путем соединения трех точек на плоскости, не принадлежащих одной прямой.

  • Общие формулы расчета площади треугольника
    • По основанию и высоте
    • Формула Герона
    • Через две стороны и угол между ними
  • Площадь прямоугольного треугольника
  • Площадь равнобедренного треугольника
  • Площадь равностороннего треугольника
    • Через длину стороны
    • Через высоту
  • Примеры задач

Общие формулы расчета площади треугольника

По основанию и высоте

Площадь (S) треугольника равняется половине произведения его основания и высоты, проведенной к нему.

Формула Герона

Для нахождения площади (S) треугольника необходимо знать длины всех его сторон. Считается она следующим образом:

p – полупериметр треугольника:

Через две стороны и угол между ними

Площадь треугольника (S) равняется половине произведения двух его сторон и синуса угла между ними.

Площадь прямоугольного треугольника

Площадь (S) фигуры равняется половине произведения его катетов.

Площадь равнобедренного треугольника

Площадь (S) рассчитывается по следующей формуле:

Площадь равностороннего треугольника

Чтобы найти площадь правильного треугольника (все стороны фигуры равны), необходимо воспользоваться одной из формул ниже:

Через длину стороны

Через высоту

Примеры задач

Задание 1
Найдите площадь треугольника, если одна из его сторон равна 7 см, а высота, проведенная к ней – 5 см.

Решение:
Используем формулу, в которой участвуют длина стороны и высота:
S = 1/2 ⋅ 7 см ⋅ 5 см = 17,5 см2.

Задание 2
Найдите площадь треугольника, стороны которого равны 3, 4 и 5 см.

Решение 1:
Воспользуемся формулой Герона:
Полупериметр (p) = (3 + 4 + 5) / 2 = 6 см.

Следовательно, S = √6(6-3)(6-4)(6-5) = 6 см2.

Решение 2:
Т.к. треугольник со сторонами 3, 4 и 5 – прямоугольный, его площадь можно посчитать по соответствующей формуле:
S = 1/2 ⋅ 3 см ⋅ 4 см = 6 см2.

ЧАЩЕ ВСЕГО ЗАПРАШИВАЮТ

Таблица знаков зодиака

Нахождение площади трапеции: формула и примеры

Нахождение длины окружности: формула и задачи

Римские цифры: таблицы

Таблица синусов

Тригонометрическая функция: Тангенс угла (tg)

Нахождение площади ромба: формула и примеры

Нахождение объема цилиндра: формула и задачи

Тригонометрическая функция: Синус угла (sin)

Геометрическая фигура: треугольник

Нахождение объема шара: формула и задачи

Тригонометрическая функция: Косинус угла (cos)

Нахождение объема конуса: формула и задачи

Таблица сложения чисел

Нахождение площади квадрата: формула и примеры

Что такое тетраэдр: определение, виды, формулы площади и объема

Нахождение объема пирамиды: формула и задачи

Признаки подобия треугольников

Нахождение периметра прямоугольника: формула и задачи

Формула Герона для треугольника

Что такое средняя линия треугольника

Нахождение площади треугольника: формула и примеры

Нахождение площади поверхности конуса: формула и задачи

Что такое прямоугольник: определение, свойства, признаки, формулы

Разность кубов: формула и примеры

Степени натуральных чисел

Нахождение площади правильного шестиугольника: формула и примеры

Тригонометрические значения углов: sin, cos, tg, ctg

Нахождение периметра квадрата: формула и задачи

Теорема Фалеса: формулировка и пример решения задачи

Сумма кубов: формула и примеры

Нахождение объема куба: формула и задачи

Куб разности: формула и примеры

Нахождение площади шарового сегмента

Что такое окружность: определение, свойства, формулы

геометрия — Нахождение площади треугольника по длинам медиан

спросил

Изменено 7 лет, 5 месяцев назад

Просмотрено 24к раз

$\begingroup$

Мой вопрос:

В треугольнике ABC длина медианы из вершины A равна $13$, длина медиана из вершины B равна $14$, длина медианы из вершины C равна $15$. Вычислите площадь треугольника ABC.

  • геометрия
  • треугольники

$\endgroup$

1

$\begingroup$

Одна стратегия, вероятно, не оптимальная, состоит в том, чтобы найти длины сторон, а затем использовать формулу Герона.

Возьмем треугольник $XYZ$ со сторонами $x$, $y$, $z$, как обычно, и пусть $m$ — длина медианы от $Z$ до стороны $XY$, длина которой $з$.

92$.

Теперь, когда у нас есть стороны, мы можем использовать формулу Герона.

$\endgroup$

$\begingroup$

Используйте формулу:

$$A = \frac{4}{3} \sqrt{s \cdot (s_m — m_1) \cdot (s_m — m_2) \cdot (s_m — m_3)} $$

где $m_1$ = 1-я медиана, $m_2$ = 2-я медиана, $m_3$ = 3-я медиана, $s_m = (m_1+m_2+m_3)/2$

Это один из самых простых методов

С уважением Джей Шарма

$\endgroup$

$\begingroup$

Площадь: $$\Delta=\dfrac 43\sqrt {s(s-a)(s-b)(s-c)}$$ Где $a,b$ и $c$ — средние длины, а $s$ — полупериметр.

$\endgroup$

1

Зарегистрируйтесь или войдите в систему

Зарегистрируйтесь с помощью Google

Зарегистрироваться через Facebook

Зарегистрируйтесь, используя электронную почту и пароль

Опубликовать как гость

Электронная почта

Требуется, но никогда не отображается

Опубликовать как гость

Электронная почта

Требуется, но не отображается

Нажимая «Опубликовать свой ответ», вы соглашаетесь с нашими условиями обслуживания, политикой конфиденциальности и политикой использования файлов cookie

геометрия — отношение площади треугольника к площади, образованной его медианами

спросил

Изменено 2 года, 6 месяцев назад

Просмотрено 2к раз

$\begingroup$

Каково отношение площади треугольника $ABC$ к площади треугольника, длины сторон которого равны медианам треугольника $ABC$?

Я вижу очевидный метод грубой силы, в котором я могу наложить на фигуру систему координат. Но есть ли лучшее решение?

  • геометрия
  • конкурс-математика

$\endgroup$

2

$\begingroup$

Пусть $X$, $Y$ и $Z$ — середины сторон. Постройте параллелограмм PYBZ и подключите PC.

По построению PY = BZ = AZ, PY || AZ, а затем PYZA также являются параллелограммом, что приводит к AP || ЗЫ || XC и AP = XC, и, в свою очередь, параллелограмм APCX. Таким образом, PC = AX и стороны треугольника PCZ являются медианами треугольника ABC.

Параллелограмм PYZA также дает AQ = QY = $\frac14$AC и затем

$$\frac{Area_{PCZ}}{Area_{ABC}}=\frac{Area_{QCZ}}{Area_{AZC} }=\frac{QC}{AC} = \frac34 $$

$\endgroup$

$\begingroup$

Площадь треугольника с учетом его медиан равна $(4/3)\sqrt{m(m-m_a)(m-m_b)(m-m_b)}$ а площадь треугольника, образованного медианами первого треугольника, по формуле Герона равна $\sqrt{m(m-m_a)(m-m_b)(m-m_b)}$.